• Shuffle
    Toggle On
    Toggle Off
  • Alphabetize
    Toggle On
    Toggle Off
  • Front First
    Toggle On
    Toggle Off
  • Both Sides
    Toggle On
    Toggle Off
  • Read
    Toggle On
    Toggle Off
Reading...
Front

Card Range To Study

through

image

Play button

image

Play button

image

Progress

1/315

Click to flip

Use LEFT and RIGHT arrow keys to navigate between flashcards;

Use UP and DOWN arrow keys to flip the card;

H to show hint;

A reads text to speech;

315 Cards in this Set

  • Front
  • Back
  • 3rd side (hint)
A 14-year-old boy is hit over the right side of the head with a baseball bat. He loses consciousness for a few minutes, but recovers promptly and continues to play. One hour later he is found unconscious in the locker room. His right pupil is fixed and dilated.
What is Diagnosis? How is it Diagnosed? What is Treatment and Prognosis?
What is it? – Acute epidural hematoma (probably right). How is it diagnosed? – CT scan. Treatment? – Emergency surgical decompression (craniotomy). Good if treated, fatal within hours if it is not.
A 32-year-old male is involved in a head-on, high-speed automobile collision. He is unconscious at the site, regains consciousness briefly during the ambulance ride and arrives at the E.R. in deep coma, with a fixed, dilated right pupil. Possible Diagnoses? Test of Choice? Remember to check what? Treatment and prognosis?
Diagnoses? – acute epidural hematoma, but acute subdural is better bet.
Test? – CT scan. Also need to check cervical spine!
Treatment? – Emergency craniotomy, poor prognosis because of brain injury.
A 77-year-old man becomes “senile” over a period of three or four weeks. He used to be active and managed all of his financial affairs. Now he stares at the wall, barely talks and sleeps most of the day. His daughter recalls that he fell from a horse about a week before the mental changes began. Diagnosis? Test? Treatment & Prognosis?
What is it? – Chronic subdural hematoma (venous bleed). Test? – CT scan.
Treatment: Surgical decompression (craniotomy). Spectacular improvement expected.
A car hits a pedestrian. He arrives in the ER in coma. He has any of these symptoms(raccoon eyes… or clear fluid dripping from the nose…or clear fluid dripping from the ear…or ecchymosis behind the ear). Diagnosis? Test? Treatment?
What is it? – Base of the skull fracture.
Test? – CT scan. Needs cervical spine X-Rays.
Therapy: needs neurosurgical consult, needs antibiotics.
A 45-year-old man is involved in a high-speed automobile collision. He arrives at the ER in coma, with fixed dilated pupils. He has multiple other injuries (extremities, etc). His blood pressure is 70 over 50, with a feeble pulse at a rate of 130. What is the reason for the low BP and high pulse rate?
NOT neurological injury (Not enough room in the head for enough blood loss to cause shock). Other significant blood loss (scalp laceration), or internal: abdomen, pelvic fractures)
A 22-year-old gang member arrives in the E.R. with multiple guns shot wounds to the abdomen. He is diaphoretic, pale, cold, shivering, anxious, asking for a blanket and a drink of water. His blood pressure is 60 over 40. His pulse rate is 150, barely perceptible. Describe State? Initial Treatments?
State? – Hypovolemic shock
Treatment: Several things at one: Big bore IV lines, Foley catheter and I.V. antibiotics. exploratory lap immediately for control of bleeding, and then fluid and blood administration unless waiting on OR.
A 22-year-old gang member arrives in the E.R. with multiple gun shot wounds to the chest AND abdomen. He is diaphoretic, pale, cold, shivering, anxious, asking for a blanket and a drink of water. His blood pressure is 60 over 40. His pulse rate is 150, barely perceptible. State? If you added distended neck veins or high CVP to symptoms? If respiratory distress, tracheal deviation, and absent breath sounds with hemithorax resonant to percussion?
State? – Hypovolemic shock still the best bet, but chest GSW raises possibility of pericardial tamponade or tension pneumothorax. If JVD/high CVP think :both tamponade and tension pneumothorax; If tracheal deviation: tension pneumothorax.
A 22-year-old gang member arrives in the E.R. with multiple guns shot wounds to the chest and abdomen. He is diaphoretic, cold, shivering, anxious, asking for a blanket and a drink of water. His blood pressure is 60 over 40. His pule rate is 150, barely perceptible. He has big distended veins in his neck and forehead. He is breathing OK, has bilateral breath sounds and no tracheal deviation. Diagnosis? Testing? Treatment?
What is it? – Pericardial tamponade. Tests: No X-Rays needed, this is a clinical diagnosis!. Treatment: Pericardial window. If positive, follow with thoracotomy, and then exploratory lap.
Single gunshot wound in the precordial area. Concerning for? Treatment?
Concern: strongly suggests pericardial tamponade, Treatment: emergency thoracotomy might be done right away without prior pericardial window.
A 22-year-old gang member arrives in the E.R. with multiple gun shot wounds to the chest and abdomen. He has labored breathing is cyanotic, diaphoretic, cold and shivering. His blood pressure is 60 over 40. His pulse rate is 150, barely perceptible. He is in respiratory distress, has big distended veins in his neck and forehead, his trachea is deviated to the left, and the right side of his chest is tympantic, with no breath sounds. Diagnosis? Management? Test?
What is it? – Tension pneumothorax. Management: Immediate big bore IV catheter into the right pleural space, followed by chest tube to the right side, STAT! Exploratory lap will follow. Test: Do NOT wait on X-Ray, this is a clinical diagnosis, and patient needs that chest tube now.
A 72 year old man who lives alone calls 911 saying that he has severe chest pain. He cannot give a coherent history when picked up by the EMT, and on arrival at the ER he is cold and diaphoretic and his blood pressure is 80 over 65. He has an irregular, feeble pulse at a rate of 130. His neck and forehead veins are distended and he is short of breath. Most likely State? management?
State: no trauma, old man, chest pain equals cardiogenic shock, from massive MI. Management: Evaluate CVP, EKG, enzymes, coronary care unit etc. Do not drown him with fluid “resuscitation”, but use thrombolytic therapy if offered.
A 17 year old girl is stun by a swarm of bees…or a man of whatever age breaks out with hives after a penicillin injection…or a patient undergoing surgery under spinal anesthetic…eventually develop BP of 75 over 25, pulse rate of 150, but they look warm and flush rather than pale and cold. CVP is low. State? Management?
What is it? – Vasomotor shock (massive vasodilation, loss of vascular tone)
Management: vasoconstrictors. Note Volume replacement would not hurt.
A 25-year-old man is stabbed in the right chest. He is moderately short of breath, has STABLE vital signs. No breath sounds on the right. Resonant to percussion. Diagnosis? Test to Verify? Treatment?
What is it? – Plain pneumothorax.
How is diagnosis verified? There is time to get a CXR.
Treatment: Chest tube to underwater seal and suction, high in the pleural cavity.
A 25-year-old man is stabbed in the right chest. He is moderately short of breath, has STABLE vital signs. No breath sounds at the base of the right chest, faint distant breath sounds at the apex. DULL to percussion. Diagnosis? Test? Treatment?
What is it? – Sounds more like hemothorax than pneumothorax. Test? - Chest X-Ray
Treatment is chest tube on the right, at the base of the pleural cavity.
A 25-year-old man is stabbed in the right chest. He is moderately short of breath, has stable vital signs. No breath sounds on at the base on the right chest, faint distant breath sounds at the apex. Dull to percussion. A chest tube placed at the right pleural base recovers 120 cc of blood, drains another 20 c in the next hour. Next Treatment Step?
Further treatment: most hemothoraxes do NOT need exploratory surgery. Bleeding is from lung parenchyma (low pressure), stops by itself. Chest tube is all that is needed. Key clue: little blood retrieved, even less afterwards.
A 25-year-old man is stabbed in the right chest. He is moderately short of breath, has blood pressure is 95 over 70, pulse rate of 100. No breath sounds at the base on the right chest, faint distant breath sounds at the apex. Dull to percussion. A chest tube placed at the right pleural base recovers 1250 cc of blood, followed by another 420 cc in the next hour. Next Step in Treatment?
Further treatment: Rare bleed from a systemic vessel (usu. intercostal). Will need thoracotomy to ligate the vessel.
A 25-year-old man is stabbed in the right chest. He is moderately short of breath, has stable vital signs. No breath sounds on the right. Resonant to percussion at the apex of the right chest, dull at the base. Chest X-Ray shows one single, large air-fluid level. Diagnosis? Treatment?
What is it? – Hemo-pneumothorax. Treatment: Chest tube, surgery only if bleeding a lot.
A 33-year-old lady is involved in a high-speed automobile collision. She arrives at the E.R. gasping for breath, cyanotic at the lips, with flaring nostrils. There are bruises over both sides of the chest, and tenderness suggestive of multiple fractured ribs. Blood pressure is 60 over 45. Pulse rate 160, feeble. She has distended neck and forehead veins, is diaphoretic. Left hemithorax has no breath sounds, is tympanitic to percussion. Diagnosis? Cause? Management? Further tests?
What is it? tension pneumothorax. Cause: The fractured ribs can act as a penetrating weapon.
Management: chest tube to the left right away! Tests? AFTER chest tube get X-Rays to rule out wide mediastinum (aortic rupture)
A 54-year-old lady crashes her car against a telephone pole at high speed. On arrival at the E.R. she is in moderate respiratory distress. She has multiple bruises over the chest, and multiple site of point tenderness over the ribs. X-Rays show multiple rib fractures on both sides. On closer observation it is noted that a segment of the chest wall on the left side caves in when she inhales, and bulges out when she exhales. Diagnosis? Management? High risk for?
What is it? – paradoxical breathing equals flail chest, underlying pulmonary contusion. Management: 1. Rule out other injuries (aortic rupture, abdominal injuries) treatment is controversial, including fluid restriction, diuretics, use of COLLOID rather than crystalloid fluids when needed, and RESPIRATORY SUPPORT. Wrong treatment is mechanically STABILIZING the part of the chest wall that moves the wrong way. High risk: to develop tension pneumothorax
A 54-year-old lady crashes her car against a telephone pole at high speed. On arrival at the E.R. she is breathing well. She has multiple bruises over the chest and multiple sites of point tenderness over the ribs. X-Rays show multiple rib fractures on both sides, but the lung parenchyma is clear and both lungs are expanded. TWO DAYS LATER her lungs “white out” on X-Rays and she is in respiratory distress. Diagnosis? Management?
What is it? – Pulmonary contusion. It does not always show up right away, may become evident one or two days after the trauma.
Management: Fluid restriction (using colloid), diuretics, respiratory support. The later is key, with intubation, mechanical ventilation and PEEP if needed.
A 54-year-old lady crashes her car against a telephone pole at high speed. On arrival at the E.R. she is breathing well. She has multiple bruises over the chest, and is exquisitely tender over the sternum at a point where there is a crunching feeling of crepitation elicited by palpation. Diagnosis? High risk for? Further Tests?
What is it? – sternal fracture. High Risk: myocardial contusion and aortic rupture.
Further tests: MI workup: EKG, cardiac enzymes, plus CT scan, transesophageal echo or arteriogram looking for aortic rupture.
A 53-year-old man is involved in a high-speed automobile collision. He has moderate respiratory distress. Physical exam shows no breath sounds over the entire left chest. Percussion is unremarkable. Chest X-Ray shows air fluid levels in the left chest. Possible NGT curling up left chest Diagnosis? Further Tests? Treatment?
What is it? – traumatic diaphragmatic rupture (always left)
Further test? Not really needed. Management: Surgical repair
A motorcycle daredevil attempts to jump over the 12 fountains in front of Caesar’s Palace Hotel in Las Vegas. As he leaves the ramp at very high speed his motorcycle turns sideways and he hits the retaining wall at the other end, literally like a rag doll. At the Er. he is found to be remarkably stable, although he has multiple extremity fractures. A chest X-Ray shows fracture of the left first rib and widened mediastinum. Diagnosis? Test of Choice? Treatment
What is it? - traumatic rupture of the aorta:
How is the diagnosis made? – Arteriogram (aortogram).
Treatment: Emergency surgical repair.
A 34-year-old lady suffers severe blunt trauma in a car accident. She has multiple injuries to her extremities, has head trauma and has a pneumothorax on the left. Shortly after initial examination it is noted that she is developing progressive subcutaneous emphysema all over her upper chest and lower neck. Diagnosis? Confirmatory Test? Management?
What is it? – Traumatic rupture of the trachea or major bronchus.
Additional findings: Chest X-Ray. Management: Fiberoptic bronchoscopy to confirm diagnosis and level of injury and to secure an airway. Surgical repair after that.
A 26-year-old lady has been involved in a car wreck. She has fractures in upper extremities, facial lacerations and no other obvious injuries. Chest X-Ray is normal. Shortly thereafter she develops hypotension, tachycardia and dropping hematocrit. Her CVP is low. What is It? Amount lost necessary for Hypovolemic Shock? How is it diagnosed? Management?
What is it? – Blood loss, most likely in the abdomen. Hypovolemic shock: one has to lose 25 to 30% of blood volume, which in the average size adult will be nearly a liter and a half. How is it diagnosed? The old, invasive way was the diagnostic peritoneal lavage. The newer, non-invasive ways for the non CRASHING patient are the CAT scan or sonogram.
Management: If stable, observation with serial CT scans will follow. If not, exploratory laparotomy.
A 19 year old gang member is shot in the abdomen with a 38 caliber revolver. The entry wound is in the epigastrium, to the left of the midline. The bullet is lodged in the psoas muscle on the right. He is hemodynamically stable, the abdomen is moderately tender. Tests and Management? Prep Prior to OR?
Management: No diagnostic tests are needed,a penetrating wound of the abdomen gets exploratory laparotomy every time. Prep:indwelling bladder catheter, a big bore venous line for fluid and a dose of broad spectrum antibiotics.
A 19 year old gang member is shot once with a 38 caliber revolver. The entry wound is in the left mid-clavicular line, two inches below the nipple. The bullet is lodged in the left paraspinal muscles. He is hemodynamically stable, but he is drunk and combative and physical exam Is difficult to do. Tests and Management?
Mangament: Must consider wound as possibly chest or abdominal since it begins at the nipple line. Therefore needs penetrating chest wound workup :chest X-Ray, chest tube if needed, plus the exploratory lap for possible abdominal.
A 27 year old intoxicated man smashes his car against a tree. He is tender over the left lower chest wall. Chest X-Ray shows fractures of the 8th, 9th and 10th ribs on the left. He has a blood pressure of 85 over 68 and a pulse rate of 128. Given level and side of rib fracture, what is possibly wrong? Tests and Management?
What is it? ruptured spleen. Note in the absence of other clues, clinically significant hidden intra-abdominal bleeding comes from a ruptured spleen.
Tests and Management: if he responds promptly to fluids and does not require blood, go for a CT scan. Further management will be continued observation with serial CT scans. If he is “crashing”, he will need the peritoneal lavage or sonogram followed by exploratory laparotomy.
A 27 year old intoxicated man smashes his car against a tree. He is tender over the left lower chest wall. Chest X-Ray shows fractures of the 8th, 9th and 10th ribs on the left. He has a blood pressure of 85 over 68 and a pulse rate of 128, which do not respond satisfactorily to fluid and blood administration. He has a positive peritoneal lavage and at exploratory laparotomy a ruptured spleen is found What is the surgical plan? Further management?
What is the issue here? – an effort will be made to repair it rather than remove it, especially in children. Further management includes administration of pneumovax and some would also immunize for hemophilus influenza B and Meningococcus.
A 31 year old lady smashes her car against a wall. She has multiple injuries including upper and lower extremity fractures. Her blood pressure is 75 over 55, with a pulse rate of 110. On physical exam she has a tender abdomen, with guarding and rebound on all quadrants. Diagnosis? Management?
What is it? – Blood in the belly CAN elicit peritoneal reaction. Management: exploratory lap, no need for tests.
A 31 year old lady smashes her car against a wall. Her hollow viscera have possibly been penetrated. What will she need?
What will she need? exploratory lap, and repair of the injuries.
A patient involved in a high speed automobile collision has multiple injuries, including a pelvic fracture. On physical exam there is blood in the meatus. Complication? Evaluation?
What is it? – pelvic fracture plus blood in the meatus means either bladder or urethral injury. Evaluation: starts with a retrograde urethrogram because urethral injury would be COMPOUNDED by insertion of a Foley catheter.
A 19 year old male is involved in a severe automobile accident. Among many other injuries he has a pelvic fracture. He has blood in the meatus, scrotal hematoma and the sensation that he wants to urinate but can not do it. Rectal exam shows a “high riding prostate”. Complication? Tests? Management?
What is it? Complete description of a posterior urethral injury.
How is the diagnosis made? retrograde urethrogram
Management: suprapubic catheter, and the repair is delayed 6 months.
A 19 year old male is involved in a motorcycle accident. Among many other injuries he has a pelvic fracture. He has blood in the meatus and scrotal hematoma. Retrograde urethrogram shows an anterior urethral injury. Management?
Mangagement: anterior urethral injuries are repaired right away.
A patient involved in a high speed automobile collision has multiple injuries, including a pelvic fracture. Insertion of a Foley catheter shows that there is gross hematuria. What is it? Assessment?
What is it? – if no blood was in the meatus to warn against foley, and since the latter was accomplished, urethra is intact. Therefore bladder injury.
Assessment: retrograde cystogram.
A patient involved in a high speed automobile collision has multiple injuries, including rib fractures and abdominal contusions. Insertion of a Foley catheter shows that there is gross hematuria, and retrograde cystogram is normal. What is it? next Step in Diagnosis? Further management?
What is it? – Lower injuries ruled out. The blood is from kidneys. How is the diagnosis made? – CT scan.
Further management: The rule is that traumatic hematuria does not need surgery even if the kidney is smashed. Operate only if the renal pedicle is avulsed or the patient is exsanguinating.
A 35 year old male is about to be discharged from the hospital where he was under observation for multiple blunt trauma sustained in a car wreck. It is then discovered that he has microscopic hematuria. Investigate?
Management: Gross traumatic hematuria in the adult always has to be investigated.
A 4 year old falls from his tricycle. In the ensuing evaluation he is found to have microscopic hematuria. Management? Tests?
Management: Microhematuria in kids needs to be investigated, as it often signifies congenital anomalies…particularly if the magnitude of the trauma does not justify the bleeding. Start with sonogram.
A 14 year old boy slides down a banister, not realizing that there is a big knob at the end of it. He smashes the scrotum and comes in to the E.R. with a scrotal hematoma the size of a grapefruit. Diagnostic Concern? Test? Management?
What is it? scrotal hematomas with question of if testicle is ruptured or not. How is the diagnosis made? Sonogram will tell.
Management: If ruptured, surgery will be needed. If intact, only symptomatic treatment.
A 41 year old male presents to the E.R. reporting that he slipped in the shower and injured his penis. Exam reveals a large penile shaft hematoma with normal appearing glans. What is it? Management?
What is it? – fracture of the tunica albugine including the usual cover story given by the patient. These always happen during sexual intercourse with woman on top…but patient will not say so.
Management: UROLOGIC EMERGENCY, Surgical repair is needed.
You get a phone call from a frantic mother. Her 7 year old girl spilled Drano all over her arms and legs. You can hear the girl screaming in pain in the background. Management?
Management: chemical injuries – particularly alkalis-need copious, immediate, profuse irrigation. Instruct the mother to do so right at home with tap water, for at least 30 minutes BEFORE rushing the girl to the E.R.
While trying to hook up illegally to cable TV, an unfortunate man comes in contact with a high tension electrical power line. He has an entrance burn wound in the upper outer thigh and an exit burn lower down on the same side. Management? Beware what?
Management: electrical burns are always much bigger than they appear to be. There is deep tissue destruction. The patient will require extensive surgical debridement, Patient needs lots of IV fluids, diuretics (osmotics), perhaps alkalinization of the urine. Beware: Myoglobinemia, leading to myoglobinuria and to renal failure.
A man is rescued by firemen from a burning building. On admission it is noted that he has burns around the mouth and nose, and the inside of his mouth and throat look like the inside of a chimney. Diagnosis? Test? Management?
What is it? smoke inhalation, chemical burn of the tracheobronchial tree. It happens with flame burns in an enclosed space. Test: bronchoscopy. Management: revolves around respiratory support.
A patient has suffered third degree burns to both of his arms when his shirt caught on fire while lighting the back yard barbecue. The burned areas are dry, white, leatherly anesthetic, and circumferential all around arms and forearms. Diagnosis? Concerning for? Management?
What is it? circumferential burns. Concern: The leatherly eschar will not expand, while the arc under the burn will develop massive edema, thus circulation will be cut off or if chest than breathing compromised. Management: Compulsive monitoring of peripheral pulses and capillary filling.
Escharotomies at the bedside at the first sign of compromised circulation.
A toddler is brought to the E.R. with burns on both of his buttocks. The areas are moist, have blisters and are exquisitely painful to touch. The story is that the kid accidentally pulled a pot of boiling water over himself. Diagnosis? Management?
What is it ? second degree burn. (Note that in kids third degree is deep bright red, rather than white leatherly as in the adult). Management: for the burn is silvadene cream.
An adult male who weighs “X” Kgs. Sustains second and third degree burns over back, indicating what is second degree (moist, blisters, painful) and what is third degree (white, leatherly, anesthetic). Management? How to Monitor Management? Treatment?
Management: FORMULA RULE 4cc per Kg. of body weight per percentage of burned area (up to 50). Percentage to be calculated by the rule of nines: one/ninth each for head and arms, two/ninths for each leg, four/ninths for the trunk. Give ringers lactate, pour it in so that half of the calculated dose goes in during first 8 hours. If the colloids are to be used, give them in the second day (not the first).
Monitoring: CVP below 15-20 and hourly urinary output. Treatment: After the obvious cleansing, silvadene cream for most areas, sulphamyelon where deep penetration is needed (cartilage, thick eschar), triple antibiotic ointment in the face near the eyes. Skin grafting will ensue.
A 42 year old lady drops her hot iron on her lap while doing the laundry. She comes in with the shape of the iron clearly delineated on her upper thigh. The area is white, dry, leatherly, anesthetic. Diagnosis? Treatment?
Diagnosis? - Small Third Degree Burn. Treatment: Early excision and skin grafting in OR on day 1.
A 22 year old gang leader comes to the E.R. with a small, 1 cm. DEEP SHARP cut over the knuckle of the right middle finger. He says he cut himself with a screwdriver while fixing his car. Diagnosis? Management?
Diagnosis? often a human bite from fighting, not the story told by the patient– Management: Surgical exploration by an orthopedic surgeon will be required.
A 65 year old West Texas farmer of Swedish ancestry has an indolent, raised, waxy, 1.2 cm skin mass over the bridge of the nose that has been slowly growing over the past three years. There are no enlarged lymph nodes in the head and neck. Lesion? Testing? Treatment?
Lesion? – Basal cell carcinoma. Testing: Full thickness biopsy at the edge of the lesion. Treatment: Surgical excision with clear margins, but conservative width.
A 71 year old West Texas farmer of Irish ancestry has a non-healing, indolent, punched out, clean looking 2 cm. ulcer over the left temple, that has been slowly becoming larger over the past three years. There are no enlarged lymph nodes in the head and neck. Lesion? Treatment?
Lesion? basal cell carcinoma. Treatment: Surgical excision with clear margins, but conservative width.
A blond, blue eyed, 69 year old sailor has a non-healing, indolent 1.5 cm. ulcer on the lower lip, that has been present, and slowly enlarging for the past 8 months. He is a pipe moker, and he has no other lesions or physical findings. Lesion? Test? Treatment?
Lesion? - Squamous cell carcinoma. Test? - Biopsy. Treatment: surgical resection with wider (about 1 cm.) clear margins. Local radiation therapy is another option.
A red headed 23 year old lady who worships the sun, and who happens to be full of freckles, consults you for a skin lesion on her shoulder that concerns her. She has a pigmented lesion that is asymetrical, with irregular borders, of different colors within the lesion, and measuring 1.8 cms. Lesion? Management?
Lesion? – melanoma or dysplastic nevus. Management: full thickness biopsy at the edge of the lesion, margin free local excision if superficial melanoma (Clarks’ levels one or two, or under 0.75 mm), wide local excision with 2 or 3 cm. margin if deep melanoma.
A 35 year old blond, blue eyed man left his native Minnesota at age 18, and has been living an idyllic life as a crew member for a sailing yacht charter operation in the Caribbean. He has multiple nevi all over his body, but one of them has changed recently. Lesion? Management?
Lesion? – Beware melanoma. Management: full thickness biopsy at the edge of the lesion, margin free local excision if superficial melanoma or wide local excision with 2 or 3 cm. margin if deep.
A 44 year old man has unequivocal signs of multiple liver metastasis, but no primary tumor has been identified by multiple diagnostic studies of the abdomen and chest. The only abnormality in the physical exam is a missing toe, which he says was removed at the age of 18 for a black tumor under the toenail. What was the primary?
Primary? Malignant melanoma - often having large time intervals.
A 32 year old gentleman had a Clark’s level 5, 3.4 mm. Deep, melanoma removed from the middle of his back three years ago. He now has a tumors in a weird places, like his left ventricle, his duodenum, his ischiorectal area. Are thes from his previous melanoma?
invasive melanoma (it has to be deep) metastasizes to all the usual places (lymph nodes plus liver-lung-brain-bone) but it goes to weird places.
An 18 year old lady has a firm, rubbery mass in the left breast that moves easily with palpation. Diagnosis? Test?
What is it? - Fibroadenoma.
Test? tissue biopsy, FNA preferably.
A 27 year old immigrant from Mexico has a 12 x 10 x 7 cm. mass in her left breast. It has been present for seven years, and slowly growing to it’s present size. The mass is firm, rubbery, completely movable, is not attached to chest wall or to overlying skin. There are no palpable axillary nodes. Diagnosis? Testing? Treatment?
Diagnosis? - Cystosarcoma Phyllodes. Testing: Tissue biopsy, given the size best done with core or incisional biopsy. Treatment: Margin-free resection will follow.
A 35 year old lady has a ten year history of tenderness in both breasts, related to menstrual cycle, with multiple lumps on both breasts that seem to “come and go” at different times in the menstrual cycle. Now has a firm, round, 2 cm. mass that has not gone away for 6 weeks. Diagnosis? Testing? Management?
Diagnosis? - Fibrocystic disease most likely. Testing: tissue diagnosis (i.e: biopsy) becomes impractical when there are lumps every month. Could do sonogram or mammogram along with treatment. Management: Therefore Aspiration of the cyst is the answer here. If the mass goes away and the fluid aspirated is clear, that’s all. If the fluid is bloody it goes to cytology. If the mass does not go away, or recurs she needs biopsy.
A 34 year old lady has been having bloody discharge from the right nipple, on and off for several months. There are NO palpable masses. Diagnosis? Testing? Treatment?
What is it? Intraductal papilloma. Testing: mammogram to look for non palpable masses. Treatment: possibly wish to find an resect the intraductal papilloma to provide symptomatic relief.
A 26 year old lactating mother has cracks in the nipple and develops a fluctuating, red, hot, tender mass in the breast, along with fever and leukocytosis. Diagnosis? If not lactating what would you be concerned for? Management?
What is it? - Abscess. Non lactating: cancer until proven otherwise. Management: Incision and drainage, with possible biopsy.
A 49 year old has a firm, 2cm. mass in the right breast, that has been present for 3 months. Testing Strategy:
Management: You have to have tissue. Core biopsy is OK, but if negative you don’t stop there: only excisional biopsy will rule out cancer.
A 69 year old lady has a 4 cm. hard mass in the right breast, with ill defined borders, movable from the chest wall but not movable within the breast. The skin overlying the mass is retracted an has an “orange peel” appearance…or the nipple became retracted six months ago. Diagnosis? Testing?
Diagnosis? cancer of the breast. Testing? tissue biopsy.
A 72 year old lady has a red, swollen breast. The skin over the area looks like orange peel. She is not particularly tender, and it is debatable whether the area is hot or not. She has no fever or leukocytosis. Diagnosis? Testing?
What is it? inflammatory cancer of the breast or cellulitis. Testing? tissue biopsy.
A 62 year old lady has an eczematoid lesion in the areola. It has been present for 3 months and it looks to her like “some kind of skin condition” that has not improved or gone away with a variety of lotions and ointments. Diagnosis? Management?
Diagnosis? breast cancer or specifically Paget’s disease of the breast, which is a cancer under the areola.
Management: tissue biopsy.
A 42 year old lady hits her breast with a broom handle while doing her housework. She noticed a lump in that area at the time, and one week later the lump is still there. She has a 3 cm. hard mass deep inside the affected breast, and some superficial ecchymosis over the area. Possible Diagnosis?
Diagnosis? cancer until proven otherwise, trauma not often the cause of lumps.
A 58 year old lady discovers a mass in her right axilla. She has a discreet, hard, movable, 2 cm. mass. Examination of her breast is negative, and she has not enlarged lymph nodes elsewhere. Diagnosis? Testing?
Diagnosis? potential presentation for cancer of the breast or lymphoma in younger patients. Testing: mammogram and enlarged lymph node biopsy.
A 60 year old lady has a routine, screening mammogram. The radiologist reports an irregular area of increased density, with fine microcalcifications, that was not present two year ago on a previous mammogram. Next Step?
Next step? - microcalcifications are description of a malignant radiological image, so stereotactic radiologic guided core biopsy is necessary.
A 44 year old lady has a 2 cm. palpable mass in the upper outer quadrant of her right breast. A core biopsy shows infiltrating ductal carcinoma. The mass is freely movable and her breast is of normal, rather generous size. She has no palpable axillary nodes. Treatment Options?
Treatment: The standard option here is segemental resection (lumpectomy), to be followed by radiation therapy to the remaining breast, as well as axillary node dissection to help determine the need for adjuvant systemic therapy.
A 62 year old lady has a 4 cm. hard mass under the nipple and areola of her rather smallish left breast. A core biopsy has established a diagnosis of infiltrating ductal carcinoma. There are NO palpable axillary nodes. Treatment Options?
Treatment Options? Lumpectomy is an option only when the tumor is small (in absolute terms and in relation to the breast) and located where most of the breast can be spared. A simple mastectomy is the choice here with sentinel node biopsy.
A 44 year old lady shows up in the Emergency Room because she is “bleeding from the breast”. Physical exam shows a huge, fungating, ulcerated mass occupying the entire right breast, and firmly attached to the chest wall. The patient maintains that the mass has been present for only “a few weeks”, but a relative indicates that it has been there at least two years, maybe longer. Diagnosis? Management?
Diagnosis: Obviously a far advanced cancer of the breast. Management: tissue biopsy, and since tumor is inoperable, and incurable, palliation can be offered with Chemotherapy.
A 37 year old lady has a lumpectomy and axillary dissection for a 3 cm. infiltrating ductal carcinoma. The pathologist reports clear surgical margins and metastatic cancer in four out of 17 axillary nodes. Next Step?
Next Step? Extensive tumors need adjuvant systemic therapy, and the rule is that premenopausal women get chemotherapy and postmenopausal women get hormonal therapy.
A 66 year old lady has a modified radical mastectomy for infiltrating ductal carcinoma of the breast. The pathologist reports that tumor measures 4 cm. in diameter and that 7 out of 22 axillary node are positive for metastasis. The tumor is estrogen and progesterone receptor positive. Adjuvant Treatment?
Treatment: clear choice for hormonal therapy, usually Tamoxifen.
A 44 year old lady complains bitterly of severe headaches that have been present for several weeks and have not responded to the usual over-the-counter headache remedies. She is two years post-op. from modified radical mastectomy for T3, N2, M0 cancer of the breast, and she had several courses of post-op chemotherapy which she eventually discontinued because of the side effects. Concern? Testing?
Concern? – brain mets until proven otherwise. Testing:
CT scan of the brain.
A 39 year old lady completed her last course of postoperative adjuvant chemotherapy for breast cancer six months ago. She comes to the clinic complaining of constant back pain for about 3 weeks. She is tender to palpation over two well circumscribed areas in the thoracic and lumbar spine. Concern? Testing?
Concern? bone mets until proven otherwise. Testing?bone scan. If positive, X-Rays are needed to rule out benign reasons for the scan to “light up”.
A two year old has a huge, pedunculated lipoma hanging out from his right upper eyelid, and obstructing his vision on that eye. Concern? Management?
Concern: brain is wired from vision during early infancy (upto 7). Defects can lead to permanent cortical “blindness”(amblyopia). Management: surgically corrected as early as possible.
A young mother is visiting your office for routine medical care. She happens to have her 18 month old baby with her, and you happen to notice that one of the pupils of the baby is white, while the other one is black. Concern?
Concern? – EMERGENCY, see ophthalmologist TODAY! A white pupil (leukocoria) at this age can be retinoblastoma.
Your distant cousins that you have not seen for years visit you and brags about their beautiful baby with “huge, shiny eyes”. They show you a picture that indeed proves their assertion. Diagnosis? Concern?
Diagnosis? - Huge eyes in babies can be CONGENITAL GLAUCOMA. Tearing will indeed make them shine all the time. Concern? If undiagnosed, blindness will ensue.
A 53 year old lady is in the ER complaining of extremely severe frontal headache. The pain started about one hour ago, shortly after she left the movies where she watched a double feature (DARK TO LIGHT). On further questioning, she reports seeing halos around the lights in the parking lot when leaving the theater. On physical exam the pupils are mid-dilated, do not react to light, the corneas are cloudy and with a greenish hue, and the eyes feel “hard as a rock”. Diagnosis? Management? Treatment?
Diagnosis? acute glaucoma.
Management: EMERGENCY, ophthalmologist is needed STAT. Treatment: pilocarpin drops, Diamox or Mannitol.
A 32 year old lady presents in the E.R. with swollen, red, hot, tender eyelids on the left eye. She has fever and leukocytosis. When prying the eyelids open, you can ascertain that her pupil is dilated and fixed and that she has very limited motion of that left eye. Diagnosis? Management? Testing? Treatment?
Diagnosis? - Orbital cellulitis. Management: ophthalmological emergency. Testing: CT scan to assess the extent of the orbital infection. Treatment: surgical drainage.
A frantic mother reaches you on the phone, reporting that her 10 year old boy accidentally splashed Drano on his face and is screaming in pain complaining that his right eye hurts terribly. Managment?
Management: copious irrigation, mother must pry the eye open under the cold water tap at home, and irrigate for about ½ hour BEFORE she brings the kid to the hospital. Upon arrival, continue irrigation, remove solid matter, and eventually re-check pH.
A 59 year old, myopic gentleman reports “seeing flashes of light” at night, when his eyes are closed. Further questioning reveals that he also sees “floaters” during the day, that they number ten or twenty, and that he also sees a cloud at the top of his visual field. Diagnosis? Management?
Diganosis? - Retinal detachment. One or two floaters would not mean that. More than a dozen is an ominous sign, and that “cloud” at the top of the visual field is hemorrhage settling at the bottom of the eye.
Management: Another ophthalmological emergency. The retina specialist will use laser treatment to “spot weld” the retina back in place.
A 77 year old man suddenly loses sight from the right eye. He calls you on the phone 10 minutes after the onset of the problem. He reports no other neurological symptoms. Diagnosis? Management?
Diagnosis? Embolic occlusion of the retinal artery. Management: ophthalmological emergency, straight to ER.
A 55 year old man is diagnosed with type two diabetes mellitus. On questioning about eye symptoms he reports that sometimes after a heavy dinner the television becomes blurry and he has to squint to see it clearly. Cause?
Cause? the lens swells and shrinks in response to swings in blood sugar.
A 54 year old obese man gives a history of burning retrosternal pain and “heartburn” that is brought about by bending over, wearing tight clothing or lying flat in bed at night. He gets symptomatic relief from antiacids, but the disease process seems to be progressing since it started several years ago. Diagnosis? Management?
Diagnosis? Gastroesophageal reflux, Management: possibly recommend endoscopy and biopsies to assess the extent of esophagitis and potential complications.
A 54 year old obese man gives a history of burning retrosternal pain and “heartburn” that is brought about by bending over, wearing tight clothing or lying flat in bed at night. He gets symptomatic relief from antiacids, but the disease process seems to be progressing since it started several years ago. Endoscopy shows severe peptic esophagitis and Barrett’s esophagus. Management?
Management: Barrett’s is premalignant. Surgery would be recommended, probably a Nissen Fundoplication.
A 62 year old man describes severe epigastric and substernal pain that he can not characterize well. There is a history suggestive of gastroesophageal reflux, and EKG and cardiac enzymes have been repeatedly negative. Test to Confirm Reflux?
Test? - The test that you do is an acid perfusion (Bernstein) test, that reproduces the pain when the lower esophagus is irrigated with an acid solution.
A 44 year old black man describes progressive dysphagia that began 3 months ago with difficulty swallowing meat, progressed to soft foods and is now evident for liquids as well. he locates the place where food “sticks” at the lower end of the sternum. He has lost 30 pounds of weight. Diagnosis? Testing?
What is it? – A classic for carcinoma of the esophagus.
Testing? – Barium swallow first, then endoscopy and biopsies. CT scan next.
A 47 year old lady describes difficulty swallowing which she has had for many years. She says that liquids are more difficult to swallow than solids, and she has learned to sit up straight and wait for the fluids to “make it through”. Occasionally she regurgitates large amounts of undigested food. Diagnosis? Testing?
What is it? achalasia.
How do you make the diagnosis? - Manometry studies.
A 24 year old man spends the night cruising bars and drinking heavily. In the wee hours of the morning he is quite drunk and he starts vomiting repeatedly. He initially brings up gastric contents only, but eventually he vomits bright red blood. Diagnosis? Management?
What is it? - Mallory Weiss tear of the esophagogastric junction.
Management: Endoscopy to ascertain the diagnosis. Bleeding is typically arterial and brisk, but self-limiting. Photocoagulation may be used if needed.
A 24 year old man spends the night cruising bars and drinking heavily. In the wee hours of the morning he is quite drunk and starts vomiting repeatedly. Eventually he has a particularly violent episode of vomiting and he feels a very severe, wrenching epigastric and low sternal pain of sudden onset. On arrival at the E.R. one hour later he still has the pain, he is diaphoretic, has fever and leukocytosis and looks quite ill. Diagnosis? Testing/Confirmation? Treatment?
What is it? – Boerhave’s syndrome.
How do you confirm the diagnosis? - Gastrographin swallow Treatment: Emergency surgical repair.
A 55 year old man has an upper G.I. endoscopy done as an outpatient to check on the progress of medical therapy for gastric ulcer. Six hours after the procedure, he returns complaining of severe, constant, retrosternal pain that began shortly after he went home. He looks prostrate, very ill, is diaphoretic, has a temperature of 104 and respiratory rate of 30. Diagnosis? Test/Confirm? Treatment?
What is it? Instrumental perforation of the esophagus.
How do you confirm the diagnosis? - Gastrographin swallow Treatment: Emergency surgical repair.
A 72 year old man has lost 40 pounds of weight over a two or three month period. He gives a history of anorexia for several months, and of vague epigastric discomfort for the past 3 weeks. Diagnosis? Testing?
What is it? Cancer of the stomach. How do you diagnose it? Endoscopy and biopsies.
A 54 year old man has had colicky abdominal pain and protracted vomiting for several days. He has developed progressive moderate abdominal distention, and has not had a bowel movement or passed any gas for five days. He has high pitched, loud bowel sounds that coincide with colicky pain, and X-Rays that show distended loops of small bowel and air-fluid levels. Five years ago he had an exploratory laparotomy for a gunshot wound of the abdomen. Diagnosis? Management?
Diagnosis? - Mechanical intestinal obstruction, due to adhesions. Management: Nasogastric suction, I.V. fluids and careful observation.
A 54 year old man has had colicky abdominal pain and protracted vomiting for several days. He has developed progressive moderate abdominal distention, and has not had a bowel movement or passed any gas for five days. He has high pitched, loud bowel sounds that coincide with the colicky pain, and X-Rays that show distended loops of small bowel and air-fluid levels. Five years ago he had an exploratory laparotomy for a gunshot wound of the abdomen. Six hours after being hospitalized and placed on nasogastric suction and I.V. fluids, he develops fever, leukocytosis, abdominal tenderness and rebound tenderness. Diagnosis? Treatment?
Diagnosis? - He has strangulated obstruction: i.e., a loop of bowel is dying –or dead- from compression of the mesenteric blood supply.
What does he need? - Emergency surgery.
A 54 year old man has had colicky abdominal pain and protracted vomiting for several days. He has developed progressive moderate abdominal distention, and has not had a bowel movement or passed any gas for five days. He has high pitched, loud bowel sounds that coincide with the colicky pain, and X-Rays that show distended loops of small bowel and air-fluid levels. On physical exam a groin mass is noted, and he explains that he used to be able to “push it back” at will, but for the past 5 days has been unable to do so. Diagnosis? Management?
Diagnosis? – Mechanical intestinal obstruction, due to an incarcerated (potentially strangulated) hernia. Management: After suitable fluid replacement needs urgent surgical intervention.
A 55 year old lady is being evaluated for protracted diarrhea. On further questioning she gives a bizarre history of episodes of flushing of the face, with expiratory wheezing. A prominent jugular venous pulse is noted on her neck. Diagnosis? Testing?
Diagnosis? Carcinoid syndrome. Testing: Serum determinations of 5-hydroxy-indoleacetic acid.
A 22 year old man develops vague periumbilical pain that several hours later becomes sharp, severe, constant and well localized to the right lower quadrant of the abdomen. On physical examination he has abdominal tenderness, guarding and rebound to the right and below the umbilicus. He has a temperature of 99.6 and a WBC of 12,500, with neutrophilia and immature forms. Diagnosis? Treatment?
What is it? – A classic for acute appendicitis.
What does he need? – Exploratory laparotomy and appendectomy.
A 59 year old is referred for evaluation because he has been fainting at his job where he operates heavy machinery. He is pale and gaunt, but otherwise his physical exam is remarkable only 4+ occult blood in the stool. Lab studies show a hemogoblin of 5. Diagnosis? Testing? Treatment?
What is it? Cancer of the right colon. How is it diagnosed? Colonoscopy and biopsies. Treatment: Blood transfusions and eventually right hemicolectomy.
A 56 year old man has bloody bowel movements. The blood coats the outside of the stool, and has been constipated, and his stools have become of narrow caliber. Diagnosis? Testing?
What is it? Cancer of the distal, left side of the colon. How is it diagnosed? Flexible sigmoidoscopy (or Endoscopy) and biopsies.
A 77 year old man has a colonoscopy because of rectal bleeding. A villous adenoma is found in the rectum and several adenomatous polyps are identified in the sigmoid and descending colon. Which polyps are premalignant? Treatment?
Malignancy? Premalignant include, in descending order of malignant conversion: familial polyposis, Gardner’s, villous adenoma and adenomatous polyps. Benign include juvenile, Peutz-Jeghers, inflammatory and hyperplastic. Treatment: excise
A 42 year old man has suffered from chronic ulcerative colitis for 20 years. He weights 90 pounds and has had at least 40 hospital admissions for exacerbations of the disease. Due to a recent relapse, he has been placed on high dose steroids and immuran. For the past 12 hours he has had severe abdominal pain, temperature of 104 and leukocytosis. He looks ill, and “toxic”. His abdomen is tender particularly in the epigastric area, and he has muscle guarding and rebound. X-Rays show a massively distended transverse colon, and there is gas within the wall of the colon. Diagnosis? Management?
What is it? - Toxic megacolon. Management: Emergency surgery for the toxic megacolon and risk of uc cancer.
A 27 year man is recovering from an appendectomy for gangrenous acute appendicitis with perforation and periappendicular abscess. He has been receiving Clindamycin and tobramycin for seven days. Eight hours ago he developed watery diarrhea, crampy abdominal pain fever and leukocytosis. Diagnosis? Testing? Treatment?
Diagnosis? Pseudomembranous colitis from overgrowth of Clostridium Difficile.
How do you diagnose it? - stool cultures, but proctosigmoidoscopy can show a typical picture. Treatment: Stop the clindamycin, give either Vancomycin or Metronidazole, and AVOID lomotil.
A 60 year old man known to have hemorrhoids reports bright red blood in the toilet paper after evacuation. Diagnosis? Management?
What is it? Probably bleeding from internal hemorrhoids. Management: It is NOT reassurance and hemorrhoid remedies by phone, BUT cancer of the rectum has to be ruled out with proctosigmoidoscopic examination.
A 60 year old man known to have hemorrhoids complains of anal itching and discomfort, particularly towards the end of the day. He has perianal pain when sitting down and finds himself sitting sideways to avoid the discomfort. He is afebrile. Diagnosis? Management?
What is it? - External hemorrhoids. Management: It is NOT reassurance and hemorrhoid remedies by phone, BUT cancer of the rectum has to be ruled out with proctosigmoidoscopic exam.
A 23 year old lady describes exquisite pain with defecation and blood streaks on the outside of the stools. Because of the pain she avoids having bowel movements and when she finally does, the stools are hard and even more painful. Physical examination can not be done, as she refuses to allow anyone to even “spread her cheeks” to look at the anus for fear of precipitating the pain. Diagnosis? Management? Treatment?
What is it? anal fissure.
Management: cancer still has to be ruled out with Examination under anesthesia. Treatment: dilitizem, nitrates, and finally lateral internal sphincterotomy.
A 28 year old male is brought to the office by his mother. Beginning four months ago he has had three operations, done elsewhere, for a perianal fistula, but after each one the area has not healed, but actually the surgical wounds have become bigger. He now has multiple unhealing ulcers, fissures all around the anus, with purulent discharge. There are no palpable masses. Diagnosis? Management?
What is it? The perianal area has fantastic blood supply and heals beautifully even though feces bathe the wounds. When it does not, you immediately think of Crohn’s disease.
Management: You still have to rule out malignancy. A proper examination with biopsies is needed. The biopsies should diagnose Crohn’s.
A 44 year old man shows up in the E.R. at 11 PM with exquisite perianal pain. He can not sit down, reports that bowel movements are very painful, and has been having chills and fever. Physical examination shows a hot, tender, red, fluctuant mass between the anus and the ischial tuberosity. Diagnosis? Management?
What is it? ischiorectal abscess. Management: Therefore examination under anesthesia and incision and drainage to always, rule out cancer.
A 62 year old man complains of perianal discomfort, and reports that there are streaks of fecal soiling in his underwear. Four months ago he had a perirectal abscess drained surgically. Physical exam shows a perianal opening in the skin, and a cord-liked tract can be palpated going from the opening towards the inside of the anal canal. Browninsh purulent discharge can be expressed from the tract. Diagnosis? Management?
What is it? fistula in ano.
Management: First rule out cancer with proctosigmoidoscopy. Then schedule elective fistulotomy.
A 55-year old, HIV positive man, has a fungating mass growing out of the anus, and rock hard, enlarged lymph nodes on both groins. He has lost a lot of weight, and looks emaciated and ill. Diagnosis? Testing? Treatment?
What is it? Squamous cell carcinoma of the anus.
Testing? Biopsies of the fungating mass.
Treatment: Nigro protocol of pre-operative chemotherapy and radiation.
A 33 year old man vomits a large amount of bright red blood. Where could the blood be coming from? Testing?
Location? the bleeding is somewhere from the tip of the nose to the ligament of Treitz. How is the diagnosis made?: for all upper G.I. bleeding, start with endoscopy.
A 33 year old man has had three large bowel movements that he describes as made up entirely of dark red blood. The last one was 20 minutes ago. He is diaphoretic, pale, has a blood pressure of 90 over 70 and a pulse rate of 110. What should be done first to determine the site of bleeding?
The first diagnostic move here is to place a nasogastric tube.
A 33 year old man has had three large bowel movements that he describes as made up entirely of dark red blood. The last one was 20 minutes ago. He is daphoretic, pale, has a blood pressure of 90 over 70 and a pulse rate of 110. A nasogastric tube returns copius amounts of bright red blood. Location:
Location: the bleeding is somewhere from the tip of the nose to the ligament of Treitz (same as vomiting)
A 33 year old man has had three large bowel movements that he describes as made up entirely of dark red blood. The last one was 20 minutes ago. He is diaphoretic, pale, has a blood pressure of 90 over 70 and a pulse rate of 110. A nasogastric tube returns clear, green fluid without blood. Where is the bleeding? Next Step?
Location: Clear fluid, WITHOUT bile, would have exonerated the area down to the pylorus, and if there is bile in the aspirate, down to the ligament of Treitz. So, he is bleeding from somewhere distal to the ligament of Treitz. Next Step: lower endoscopy is notoriously difficult in massive bleeding. If he is bleeding at more than 2 cc. per minute, emergency angiogram.
A 72 year old man had three large bowel movements that he describes as made up entirely of dark red blood. The last one was two days ago. He is pale, but has normal vital signs. A nasogastric tube returns clear, green fluid without blood. What does this mean? Next step?
Meaning? - The clear aspirate is meaningless because not bleeding right now. Next Step? Angiography is not the first choice for slow bleeding or bleeding that has stopped. The first choice now is endoscopies, both upper and lower.
A 7 year old boy passes a large bloody bowel movement. Diagnosis? Testing?
Diagnosis? In this age group, Meckel’s diverticulum.
Testing? radioactively labeled technetium scan (not the one that tags reds cells, but the one that identifies GASTRIC MUCOSA).
A 41 year old man has been in the ICU for two weeks, being treated for idiopathic hemorrhagic pancreatitis. He has had several percutaneous drainage procedures for pancreatic abscesses, chest tubes for pleural effusions, and bronchoscopies for atelectasis. He has been in and out of septic shock and respiratory failure several times. Ten minutes ago he vomited a large amount of bright red blood, and as you approach him he vomits again what looks like another pint of blood. Diagnosis? Management?
Diagnosis? In this setting, it has to be STRESS ULCER.
Management: It should have been prevented by keeping the pH of the stomach above 4 with H2 blockers, antiacids or both; but once the bleeding takes place the diagnosis is made as usual with endoscopy. Treatment will be difficult, and it may require angiographic embolization of the left gastric artery.
A 59 year old man arrives in the E.R. at 2 AM, accompanied by his wife who is wearing curlers on her hair and a robe over her nightgown. He has abdominal pain that began about one hour ago, and is now generalized, constant and extremely severe. He lies motionless in the stretcher, is diaphoretic and has shallow, rapid breathing. His abdomen is rigid, very tender to deep palpation, and has guarding and rebound tenderness in all quadrants. Diagnosis? Managment?
What is it? acute abdomen, i.e.generalized acute peritonitis. Management: The acute abdomen DOES NOT need a precise diagnosis to proceed with surgical exploration. Lower lobe pneumonia and myocardial infarction to have to be ruled out with chest X-Ray and EKG, and it would be nice to have a normal amylase but you should do emergency exploratory laparotomy.
A 62 year old man with cirrhosis of the liver and ascitis, presents with generalized abdominal pain that started 12 hours ago. He now has moderate tenderness over the entire abdomen, with some guarding and equivocal rebound. He has mild fever and leukocytosis. Diagnosis? Testing? Treatment?
What is it? Peritonitis in the cirrhotic with ascitis, could be primary peritonitis which does not need surgery rather than the acute peritonitis secondary to an intraabdominal catastrophe that requires emergency operation.
Testing? Cultures of the ascitic fluid may yield a single organism. Treatment: will be with the appropriate antibiotics.
A 43 year old man develops excruciating abdominal pain at 8:18 PM. When seen in the E.R. at 8:50 PM, he has a rigid abdomen, lies motionless in the examining table, has no bowel sounds and is obviously in great pain, which he describes as constant. X-Ray shows free air under the diaphragms. Diagnosis? Treatment?
What is it? Acute abdomen plus perforated viscus equals perforated duodenal ulcer in most cases.
What needs to be done? Emergency exploratory laparotomy.
A 44 year old alcoholic male presents with severe epigastric pain that began shortly after a heavy bout of alcoholic intake, and reached maximum intensity over a period of two hours. The pain is constant, radiates straight through to the back and is accompanied by nausea, vomiting and retching. He had a similar episode two years ago, for which he required hospitalization. Diagnosis? Tests? Management?
What is it? Acute pancreatitis. How is it diagnosed? Serum and urinary amylase and lipase determinations. CT scan if the diagnosis is unclear.
Management: NPO, NG suction, IV fluids.
A 43 year old obese lady, mother of six children, has severe right upper quadrant abdominal pain that began six hours ago. The pain was colicky at first, radiated to the right shoulder and around towards the back, and was accompanied by nausea and vomiting. For the past 2 hours the pain has been constant. She has tenderness to deep palpation, muscle guarding and rebound in the right upper quadrant. Her temperature is 101 and she has a WBC of 16,000. She has had similar episodes of pain in the past, brought about by ingestion of fatty food, but they all had been of brief duration and relented spontaneously or with anticholinergic medications. Diagnosis? Tests? Treatment?
What is it? Acute cholecystitis. How is the diagnosis made? Sonogram should be the first choice. If equivocal, an “HIDA” scan (radionuclide excretion scan). Treatment: Medical management in most cases will “cool down” the process. Surgery will follow.
A 52 year old man has right flank colicky pain of sudden onset, that radiates to the inner thigh and scrotum. There is microscopic hematuria. Diagnosis? Tests?
What is it? Ureteral colic
How is the diagnosis made? Urological evaluation always begins with a plain film of the abdomen (a “KUB”). Nowadays sonogram often is the next step, but prior it was intravenous pyelogram (IVP).
A 59 year old lady has a history of three prior episodes of left lower quadrant abdominal pain for which she was briefly hospitalized and treated with antibiotics. Now she has left lower quadrant pain, tenderness, and a vaguely palpable mass. She has fever and leukocytosis. Diagnosis? Tests? Treatment?
What is it? Acute diverticulitis. How is the diagnosis made? CT scan.
Treatment: is medical for the acute attack (antibiotics, NPO) but elective sigmoid resection is advisable for recurrent disease. Emergency surgery (resection or colostomy) may be needed if she gets worse or does not respond to treatment.
An 82 year old man develops severe abdominal distension, nausea, vomiting and colicky abdominal pain. He has not passes any gas or stool for the past 12 hours. He has a tympanitic abdomen with hyperactive bowel sounds. X-Ray shows distended loops of small and large bowel, and a very large gas shadow that is located in the right upper quadrant and tapers towards the left lower quadrant with the shape of a parrot’s BEAK. Diagnosis? Management?
What is it? Volvulus of the sigmoid. Management: Proctosigmoidoscopy should relieve the obstruction. Rectal tube is another option. Eventually surgery to prevent recurrences could be considered.
A 79 year old man with atrial fibrillation develops an acute abdomen. He has a silent abdomen, with diffuse tenderness and mild rebound. There is a trace of blood in the rectal exam. He has acidosis and looks quite sick. X-Rays show distended small bowel and distended colon up to the middle of the transverse colon. Diagnosis? Treatment?
What is it? - Acute abdomen in the elderly who has atrial fibrillation, brings to mind embolic occlusion of the mesenteric vessels. Acidosis frequently ensues, and blood in the stool is often seen. Treatment: very little, the bowel is normally dead.
A 53 year old man with cirrhosis of the liver develops malaise, vague right upper quadrant abdominal discomfort and 20 pound weight loss. Physical exam shows a palpable mass that seems to arise from the left lobe of the liver. Alpha feto protein is significantly elevated. Diagnosis? Next Step? Treatment?
What is it? Probably liver cell carcinoma (hepatoma)
Next move? CT scan. Treatment: If confined to one lobe, resection.
A 53 year old man develops vague right upper quadrant abdominal discomfort and a 20 pound weight loss. Physical exam shows a palpable liver with nodularity. Two years ago he had a right hemicolectomy for cancer of the ascending colon. His carcinoembryogenic antigen (CEA) had been within normal limits right after his hemicolectomy, is now ten times normal. Diagnosis? Next Move? Treatment?
What is it? - Metastasis to the liver from colon cancer.
Next move? CT scan to ascertain extent. Treatment: If mets are confined to one lobe, resection may be done. Otherwise, chemotherapy.
A 24 year old lady develops moderate, generalized abdominal pain of sudden onset, and shortly thereafter faints. At the time of evaluation in the ER she is pale, tachycardic, and hypotensive. The abdomen is mildly distended and tender, and she has a hemogoblin of 7. There is no history of trauma. On inquiring as to whether she might be pregnant, she denies the possibility because she has been on birth control pills since she was 14, and has never missed taking them. Diagnosis? Management?
What is it? - Bleeding from a ruptured hepatic adenoma, secondary to birth control pills. Management: It’s pretty clear that she is bleeding into the belly, but a CAT scan will confirm it and probably show the liver adenoma as well. Surgery will follow.
A 44 year old lady is recovering from an episode of acute ascending cholangitis secondary to choledocholithiasis. She develops fever and leukocytosis and some tenderness in the right upper quadrant. A sonogram reveals a liver abscess. Diagnosis? Treatment?
This is a PYOGENIC abscess, Treatment: it needs to be drained.
A 29 year old migrant worker from Mexico develops fever and leukocytosis, as well as tenderness over the liver when the area is percussed. He has mild jaundice and an elevated alkaline phosphatase. Sonogram of the right upper abdominal area shows a normal biliary tree, and an abscess in the liver. Diagnosis? Management?
What is it? an amebic abscess…very common in Mexico. Management: Alone among abscesses, this one in most cases does not have to be drained, but can be effectively treated with Metranidazole. Get serology for amebic titers, but don’t wait for the report (it will take 3 weeks). Start the patient on Metranidazole. Prompt improvement will tell you that you are on the right tract. When the serologies come back the patient will be well and your diagnosis will be confirmed.
A 42 year old lady is jaundiced. She has a total bilirubin of 6 and the laboratory reports that the unconjugated, indirect bilirubin is 6 and the direct, conjugated bilirubin is zero. She has no bile in the urine. Diagnosis? Next Step?
What is it? hemolytic jaundice. What do you do next? Try to figure out what is chewing her red cells.
A 19 year old college student returns from a trip to Cancun, and two weeks later develops malaise, weakness and anorexia. A week later he notices jaundice. When he presents for evaluation his total bilirubin is 12, with 7 indirect and 5 direct. His alkaline phosphatase is mildly elevated, while the SGOT and SGPT (transaminases) are very high. Diagnosis? Management?
What is it? Hepatocellular jaundice. Management: Get serologies to confirm diagnosis and type of hepatitis.
A patient with progressive jaundice which has been present for four weeks is found to have a total bilirubin of 22, with 16 direct and 6 indirect, and minimally elevated SGOT. The alkaline phosphatase was twice normal value couple of weeks ago, and now is about six times the upper limit of normal. Diagnosis? Next move?
What is it? obstructive jaundice. Next move? Sonogram, looking for dilated intrahepatic ducts, possibly dilated extrahepatic ducts as well, and if we get lucky a finding of gallstones.
A 40 year old, obese mother of five children presents with progressive jaundice which she first noticed four weeks ago. She has a total bilirubin of 22, with 16 direct and 6 indirect, and minimally elevated SGOT. The alkaline phosphatase is about six times the upper limit of normal. She gives a history of multiple episodes of colicky right upper quadrant abdominal pain, brought about by ingestion of fatty food. Diagnosis? Next Step?
What is it? obstructive jaundice, with a good chance of being due to stones.
What do you do next? Start with the sonogram. If you need more tests after that, ERCP is the next move, which could also be used to remove the stones from the common duct. Cholecystectomy will eventually have to be done.
A 66 year old man presents with progressive jaundice which he first noticed six week ago. He has a total bilirubin of 22, with 16 direct and 6 indirect, and minimally elevated SGOT. The alkaline phosphatase is about six times the upper limit of normal. He has lost 10 pounds over the past two months, but is otherwise asymptomatic. A sonogram shows dilated intrahepatic ducts, dilated extrahepatic ducts and a very distended, thin walled gallbladder. Diagnosis? Next Step?
What is it? Malignant obstructive jaundice. A distended gallbladder is an ominous sign. What do you do next? You already have the sonogram. Next move is CAT scan and ERCP.
A 66 year old man presents with progressive jaundice which he first noticed six weeks ago. He has a total bilirubin of 22, with 16 direct and 6 indirect, and minimally elevated SGOT. The alkaline phosphatase is about six times the upper limit of normal. He is otherwise asymptomatic. A sonogram shows dilated intrahepatic ducts, dilated extrahepatic ducts and a very distended, thin walled gallbladder. Except for the dilated ducts, CT scan is unremarkable. ERCP shows a narrow area in the distal common duct, and a normal pancreatic duct. Diagnosis? Treatment? Next move?
What is it? Malignant, probably cholangiocarcinoma at the lower end of the common duct. Treatment: pancreatoduodenectomy Whipple
Next move: brushings of the common duct for cytological diagnosis.
A 64 year old lady presents with progressive jaundice which she first noticed two weeks ago. She has a total bilirubin of 12, with 8 direct and 4 indirect, and minimally elevated SGOT. The alkaline phosphatase is about ten times the upper limit of normal. She is otherwise asymptomatic, but is found to be slightly anemic and to have positive occult blood in the stool. A sonogram shows dilated intrahepatic ducts, dilated extrahepatic ducts and very distended, thin walled gallbladder. Diagnosis? Treatment? Next Move?
What is it? malignant. The coincidence of slowly bleeding into the GI tract at the same time that she develops obstructive jaundice points to an ampullary carcinoma. Treatment: malignancy that can be cured with radical surgery.
Next move: Endoscopy.
A 56 year old man presents with progressive jaundice which he first noticed six weeks ago. He has a total bilirubin of 22, with 16 direct and 6 indirect, and minimally elevated SGOT. He alkaline phosphatase is about eight times the upper limit of normal. He has lost 20 pounds over the past two months, and has a persistent, nagging mild pain deep into his epigastrium and in the upper back. His sister died at age 44 from a cancer of the pancreas. A sonogram shows dilated intrahepatic ducts, dilated extrahepatic ducts and a very distended, thin walled gallbladder. Diagnosis? Confirm?
What is it? Bad news. Cancer of the head of the pancreas.
How do clinch the diagnosis?: CAT scan; then ERCP –which will probably show obstruction of both common duct and pancreatic duct.
A white, fat, female, aged 40 and mother of five children gives a history of repeated episodes of right upper quadrant abdominal pain brought about by the ingestion of fatty foods, and relieved by the administration of anticholinergic medications. The pain is colicky, radiates to the right shoulder and around to the back, and is accompanied by nausea and occasional vomiting. Physical exam is unremarkable. Diagnosis? Next Move? Treatment?
What is it? Gallstones, with biliary colic. Next move: Sonogram. Treatment: Elective cholecystectomy will follow.
A 43 year old obese lady, mother of six children, has severe right upper quadrant abdominal pain that began six hours ago. The pain was colicky at first, radiated to the right shoulder and around towards the back, and was accompanied by nausea and vomiting. For the past 2 hours the pain has been constant. She has tenderness to deep palpation, muscle guarding and rebound in the right upper quadrant. Her temperature is 101 and she has a WBC of 16,000. She has had similar episodes of pain in the past, brought by ingestion of fatty food, but they all had been of brief duration and relented spontaneously or with anticholinergic medications. Diagnosis?
What is it? acute cholecystitis.
A 43 year old obese lady, mother of six children, has severe right upper quadrant abdominal pain that began three days ago. The pain was colicky at first, but has been constant for the past two and a half days. She has tenderness to deep palpation, muscle guarding and rebound in the right upper quadrant. She has temperature spikes to 104 and 105, with chills. Her WBC is 22, 000, with a shift to the left. Her bilirubin is 5 and she has an alkaline phosphatase of 2,000 (about 20 times normal). She has had episodes of colicky pain in the past, brought about by ingestion of fatty food, but they all had been of brief duration and relented spontaneously or with anticholinergic medications. Diagnosis? Next Test? Management?
What is it? Acute ascending cholangitis. Further test?: The diagnosis is already clear. Sonogram might confirm dilated ducts.
Management: This is an emergency decompression of the biliary tract. To achieve the latter ERCP is the first choice, but PTC (percutaneous transhepatic cholangiogram) is another option.
A white, fat, female, aged 40 and mother of five children gives a history of repeated episodes of right upper quadrant abdominal pain brought about by the ingestions of fatty foods, and relieved by the administration of anticholinergic medications. The pain is colicky, radiates to the right shoulder and around to the back, and is accompanied by nausea and occasional vomiting. This time she had a shaking chill with the colicky pain, and the pain lasted longer than usual. She has mild tenderness to palpation in the epigastrium and right upper quadrant. Laboratory determinations show a bilirubin of 3.5, an alkaline phosphatase 5 times normal and a serum amylase 3 times normal value. Diagnosis? Management? Treatment?
What is it? - She passed a common duct stone and had a transient episode of cholangitis (the shaking chill, the high phosphatase) and a bit of biliary pancreatitis (the high amylase). Treatment: What does she need?: start with sonogram. It will confirm the diagnosis of gallstones. Treatment: If she continues to get well, elective cholecystectomy will follow. If she deteriorates, she may have the stone still impacted at the Ampulla of Vater, and may need ERCP and sphincterotomy to extract it.
A 33 year old, alcoholic male, shows up in the E.R. with epigastric and mid-abdominal pain that began 12 hours ago shortly after the ingestion of a large meal. The pain is constant, very severe, and it radiates straight through to the back. He vomited twice early on, but since then has continued to have retching. He has tenderness and some muscle guarding in the upper abdomen, is afebrile and has mild tachycardia. Serum amylase is 1200, and his hematocrit is 52. Diagnosis? management?
What is it? – Acute edematous pancreatitis. Management: put the pancreas at rest: NPO, NG suction, IV fluids.
A 56 year old alcoholic male is admitted with a clinical picture of acute upper abdominal pain. The pain is constant, radiates straight thorugh to the back, and is extremely severe. He has a serum amylase of 800, WBC of 18,000 blood glucose of 150, serum calcium of 6.5 and a hematocrit of 40. He is given IV fluids and kept NPO with NG suction. By the next morning, his hematocrit has dropped to 30 the serum calcium has remained below 7 in spite of calcium administration, his BUN has gone up to 32 and he has developed metabolic acidosis and a low arterial PO2. Diagnosis? Management?
What is it? hemorrhagic pancreatitis. What do you do? Very intensive support, beware pancreatic abscesses that need to be drained as soon as they appear. Thus SERIAL CT scans will be required.
A 57 year old alcoholic male is being treated for acute hemorrhagic pancreatis. He was in the intensive care unit fore one week, required chest tubes for pleural effusion, and was on a respirator for several days, but eventually improved enough to be transferred to the floor. Two weeks after the onset of the disease he begins to spike fever and to demonstrate leukocytosis. Diagnosis? Next Test? Treamtent?
What is it? Pancreatic abscess. How do we confirm it? CT scan. What does he need? Drainage.
A 49 year old alcoholic male presents with ill-defined upper abdominal discomfort and early satiety. On physical exam he has a large epigastric mass that is deep within the abdomen, and actually hard to define. He was discharged from the hospital 5 weeks ago, after successful treatment for acute pancreatitis. Diagnosis? Next Test? Treatment?
What is it? Pancreatic pseudocyst. Next Test: You could diagnose it on the cheap with a sonogram, but CT scan is probably the best choice. Treatment: It will need to be drained, with CT guidance.
A 55 year old lady presents with vague upper abdominal discomfort, early satiety and a large but ill-defined epigastric mass. Five weeks ago she was involved in an automobile accident where she hit the upper abdomen against the steering wheel. Diagnosis? Next Test?Management?
What is it? pancreatic pseudocyst, secondary to trauma rather than as a sequela of pancreatitis. Next Test: You could diagnose it on the cheap with a sonogram, but CT scan is probably the best choice. Treatment: It will need to be drained, with CT guidance.
A disheveled, malnourished individual shows up in the emergency room requesting medication for pain. He smells of alcohol and complains bitterly of constant epigastric pain, radiating straight through to the back that he says he has had for several years. He has diabetes, steatorrhea and calcifications in the upper abdomen in a plain X-Ray. Diagnosis? Management?
Diagnosis? Chronic pancreatitis. Management:
Stopping the alcoholic intake is the first step. Replacement of pancreatic enzymes and control of the diabetes are obvious needs.
On the first post-operative day after an open cholecystectomy, a patient has a temperature of 101. What is most likely cause? Management?
What is it? Atelectasis.
Management: Listen to the chest, chest X-Ray, encourage deep breathing and coughing.
On the third post-operative day after an open cholecystectomy, a patient develops a temperature of 101. What is most likely cause? Management?
What is it? Urinary tract infection. Management: Urinalysis, Urinary culture, appropriate antibiotics.
On the fourth post-operative day after an open cholecystectomy, a patient develops a temperature of 101. There is tenderness to deep palpation in the calf, particularly when the foot is dorsiflexed. Diagnosis? Testing? Treatment?
What is it? Deep venous thrombosis. Next Test: Duplex ultrasound to confirm diagnosis. Treatment: Anticagulation to prevent thrombus propagation.
Seven days after an inguinal hernia repair, a patient returns to the clinic because of fever. The wound is red, hot and tender. Diagnosis? Management?
What is it? Wound infection.
Management: Open the wound, drain the pus, pack it open.
Two weeks after an open cholecystectomy a patient develops fever and leukocytosis. The wound is healing well and does not appear to be infected. Diagnosis? Management?
What is it? A deep abscess, prime suspects are subphrenic or subhepatic lxn. Management: CT scan to find the abscess and to guide the radiologist for the percutaneous drainage.
On the fifth post-operative day after a right hemicolectomy for cancer, the dressings covering the midline abdominal incision are found to be soaked with a clear, pinkish, SALMON-colored fluid. Diagnosis? Management?
What is it? Wound dehiscence.
Management: Keep the patient in bed, tape his belly together and schedule surgery for re-closure of the wound if the patient can take the re-operation. If too sick, the development of a ventral incisional hernia may have to be accepted now and repaired later. If the patient gets out of bed, or sneezes forcefully, you may be confronted evisceration of small bowel and in that case, keep the bowel covered and moist with sterile dressings, and rush the patient to the OR for re-closure.
A 62 year old lady was drinking her morning cup of coffee at the same time she was applying her makeup, and she noticed in the mirror that there was a lump in the lower part of her neck, visible when she swallowed. She consult you for this and on physical exam you ascertain that she indeed has a dominant, 2 cm. mass on the left lobe of her thyroid as well as two smaller masses on the right lobe. They are all soft and she has no palpable lymph nodes in the neck. Next Step?
Next Step: Most thyroid nodules are benign, so FNA, select cases may go straight to surgical removal.
A 21 year old college student is found on a routine physical examination to have a singe, 2 cm. nodule in the thyroid gland. The young man had radiation to his head and neck when he was thirteen years old because of persistent acne. His thyroid function tests are normal. Management?
Management: This one will need surgery, but if offered FNA is still your first answer.
A 44 year old lady has a palpable mass in her thyroid gland. She also describes losing weight in spite of a ravenous appetite, palpitations and heat intolerance. She is a thin lady, fidgety and constantly moving, with moist skin and a pulse rate of 105. Diagnosis? Testing? Treatment?
What is it? active adenoma causing hyperthyroidism.
Testing: confirm hyperthyroidism by measuring free T4. Confirm source of the excessive hormone with radioactive iodine scan. Treatment: Do surgery after BETA BLOCKING.
A 22 year old male has a 2 cm. round firm mass in the lateral aspect of his neck, which has been present for four months. Clinically this is assumed to be an enlarged jugular lymph node and it is eventually removed surgically. The pathologist reports that the tissue removed is normal thyroid tissue. Diagnosis? Testing? Management?
What is it? There is no such thing as “lateral aberrant thyroid”. This is metastatic follicular carcinoma from an occult primary in the thyroid gland.
Testing: Look for the primary with a thyroid scan. Management: surgery.
An automated blood chemistry panel done during the course of a routine medical examination indicates that an asymptomatic patient has a serum calcium of 12.1 in a lab where the upper limit of normal is 9.5. Repeated determinations are consistently between 10.5 and 12.6. Serum phosphorus is low. Most likely Diagnosis? Testing? Treatment?
What is it? Parathyroid adenoma. Testing? PTH determination and sistimibi scan to localize the adenoma. Treatment: Surgery
A 32 year old woman is admitted to the psychiatry unit because of wild mood swings. She is found to be hypertensive and diabetic and to have osteoporosis. (she had not been aware of such diagnosis beforehand). It is also ascertained that she has been amenorrheic and SHAVING for the past couple of years. She has gross centripetal obesity, with moon fascies and Buffalo hump, and thin, bruised extremities. A picture from 3 years ago shows a person of very different, more normal appearance. Diagnosis? Testing? Treatment?
What is it? Cushings.
Testing? Start with AM and PM cortisol determinations. Later she will get dexamethasone suppression tests and MRI of the head looking for the pituitary microadenoma. Treatment: surgery by the trans-nasal, trans-sphenoidal route.
A 28 year old lady has virulent peptic ulcer disease. Extensive medical management including eradication of H.Pylori fails to heal her ulcers. She has several duodenal ulcers in the first and second portions of the duodenum. She has watery diarrhea. Diagnosis? Testing? Treatment:
What is it? - Gastrinoma (Zollinger-Ellison).
Testing? Start by measuring serum gastrin. Later CT scans of the pancreas looking for the tumor. Treatment: surgical removal.
A second year medical student is hospitalized for a neurological work-up for a seizure disorder of recent onset. During one of his convulsions it is determined that his blood sugar is extremely low. Further work-up shows that he has high levels of insulin in the blood with low levels of C-peptide. Diagnosis? Management?
What is it? Exogenous administration of insulin.
Management: psychiatric evaluation and counseling. If it had been insulinoma, CT scan or MRI looking for the tumor in the pancreas, to be subsequently removed surgically.
A 48 year old lady has had severe, migratory necrolytic dermatitis for several years, unresponsive to all kinds of “herbs and unguents”. She is thin, has mild stomatitis and mild diabetes mellitus. Diagnosis? Testing? Treatment?
Diagnosis? Glucagonoma. Testing? Determine glucagon levels. Eventually CT scan or MRI looking for the tumor in the pancreas. Treatment: Surgery. If inoperable, Somatostatin can help symptomatically and Streptozocin is the indicated chemotherapeutic agent.
A 45 year old lady comes to your office for a “regular checkup”. On repeated determinations you confirm the fact that she is hypertensive. When she was in your office three years ago, her blood pressure was normal. Laboratory studies at this time show a serum sodium of 144 mEq/L, a serum bicarbonate of 28 mEq/L, and a serum potassium concentration of 2.1 mEq/L. The lady is taking no medications of any kind. Diagnosis? Testing? Treatment?
Diagnosis? Hyperaldosteronism via possible adenoma or hyperplasia. Testing? Start with determinations of aldosterone and renin levels. If confirmatory (aldo high, renin low) proceed with determinations lying down and sitting up, to differentiate hyperplasia (not surgical) from adenoma (surgical). Treatment: hyperplasia with aldactone. Adenoma get imaging studies (CT scan or MRI) and surgery.
A thin, hyperactive 38 year old lady is frustrated by the inability of her physicians to help her. She has episodes of severe pounding headache, with palpitations, profuse perspiration and pallor, but by the time she gets to her doctor’s office she checks out normal in every respect. Diagnosis? Testing? Treatment?
What is it? pheochromocytoma.
Testing? -24 hr. urinary determination of metanephrine and VMA (Vanillylmandelic acid). Follow with CT scan of adrenal glands. Treatmetn: Surgery will eventually be done, with careful pharmacological preparation with ALPHA-blockers.
A 17 year old man is found t have a blood pressure of 190/115. This is checked repeatedly in both arms and it is always found to be elevated, but when checked in the legs it is found to be normal. Diagnosis? Testing? Treatment?
What is it? Coarctation of the aorta. Further testing; start with a chest X-Ray, looking for scalloping of the ribs. Eventually aortogram. Treatment: surgery.
A 23 year old lady has had severe hypertension for two years, and she does not respond well to the usual medical treatment for that condition. A bruit can be faintly heard over her upper abdomen. Diagnosis? Management?
What is it? Renovascular hypertension due to fibromuscular dysplasia.
Management: Eventually arteriogram will precede surgical correction (or balloon dilatation).
A 72 year old man with multiple manifestations of arteriosclerotic occlusive disease has hypertension of RELATIVELY RECENT ONSET, and refractory to the usual medical therapy. He has a faint bruit over the upper abdomen. Diagnosis?
What is it? Renovascular hypertension due to arteriosclerotic plaque at the origin of the renal artery…or arteries (this is usually bilateral).
Within eight hours after birth, it is noted that a baby has excessive salivation. A small, soft nasogastric tube is inserted and the baby is taken to X-Ray to have a “babygram” done. The film shows the tube coiled back upon itself in the upper chest. There is air in the gastrointestinal tract. Diagnosis? Look For What? what Tests? Treatment?
What is it? Tracheo-esophageal fistula, the most common type with proximal blind esophageal pouch and distal TE fistula. Look for: the associated anomalies (“VACTER”: vertebral, anal, cardiac, TE and renal/radial). Next Tests: vertebral and radial will be seen in the same X-ray you already took, you need ECHO for the heart, Sonogram for the kidneys and exam for the anus. Treatment: surgery.
A newborn baby is found on physical exam to have an imperforate anus. What to look for?
Look for: the “VACTER”, for the imperforate anus, look for a fistula nearby (to the vagina in little girls, to the perineum in little boys), which will help determine the level of the blind pouch and the timing and type of surgery (primary repair versus colostomy and repair later).
A newborn baby is noted to be tachypneic, cyanotic and grunting. The abdomen is scaphoid and there are bowel sounds heard over the left chest. An X-Ray confirms that there is bowel in the left thorax. Shortly thereafter, the baby develops significant hypoxia and acidosis. Diagnosis? Management and Treatment?
What is it? Congenital diaphragmatic hernia.
Management: The main problem is the hypoplastic lung. It is better to wait 36 to 48 hours to do surgery to allow transition from fetal circulation to newborn circulation. Meanwhile the trick is to keep the kid alive with endotracheal intubation, hyperventilation (careful not to blow up the other lung), sedation and NG suction.
At the time of birth it is noted that a child has a large abdominal wall defect to the right of the umbilicus. There is a normal cord, but protruding from the defect there is a matted mass of angry looking, edematous bowel loops. Diagnosis? Management?
What is it? Gastroschisis.
(distinguish gastroschisis from omphalocele) correct. Management: surgery to put bowel back inside.
A newborn baby is noted to have a shiny, thin, membranous sac at the base of the umbilical cord. Inside the sac one can see part of the liver, and loops of normal looking bowel. Diagnosis? Management?
What is it? omphalocele.
Management: look for other congenital defects, then surgical repair.
A newborn is noted to have a moist medallion of mucosae occupying the lower abdominal wall, above the pubis and below the umbilicus. It is clear that urine is constantly bathing this congential anomaly. Diagnosis? Concern?
What is it? Exstrophy of the urinary bladder.
Concern? TRANSFER NOW for repair by Specialist hospital within the first 48 hours, otherwise it will not have a good chance to succeed.
Half an hour after the first feed, a baby vomits greenish fluid. The mother had polyhydramnius and the baby has Down’s syndrome. X-Ray shows a “double bubble sign”: a large air fluid level in the stomach, and smaller one in the first portion of the duodenum. There is no gas in the rest of the bowel. Diagnosis? Managment?
What is it? The two conditions that this could be are duodenal atresia and annular pancreas.
Management: with complete obstruction, emergency surgery will be needed, but these kids have lots of other congenital anomalies, look for them first.
Half an hour after the first feed, a baby vomits greenish fluid. X-Ray shows a double bubble sign”: a large air fluid level in the stomach, and a smaller one in the first portion of the duodenum. There is air in the distal bowel, beyond the duodenum, in loops that are not distended. Diagnosis? Management? Next Test?
What is it? three choices: it could be incomplete obstruction from duodenal stenosis or annular pancreas, or it could be malrotation. Next Test? Do a contrast enema, and if not diagnostic order a gastrographin upper GI study. Note that Malrotation is a super emergency, bowel could necrose.
A newborn baby has repeated green vomiting during the first day of life, and does not pass any meconium. Except for abdominal distention, the baby is otherwise normal. X-Ray shows multiple air fluid levels and distended loops of bowel. Diagnosis? Management?
What is it? Intestinal atresia. Management: This one is due to a vascular accident in uteru, thus there are no other congenital anomalies to look for, but there may be multiple points of atresia.
A very premature baby develops feeding intolerance, abdominal distention and a rapidly dropping PLATELET count. The baby is four days old, and was treated with indomethacin for a patent ductus. Diagnosis? Management? Treatment?
What is it? Necrotizing enterocolitis. Management: Stop all feedings, broad spectrum antibiotics, IV fluids/nutrition. Treatment: Surgical intervention if they develop abdominal wall erythema, air in the biliary tree or pneumoperitoneum.
A three day old, full term baby is brought in because of feeding intolerance and bilious vomiting. X-Ray shows multiple dilated loops of small bowel and a “GROUND GLASS” appearance in the lower abdomen. The mother has cystic fibrosis. Diagnosis? Testing? Treatment?
What is it? Meconium ileus.
Testing: Gastrografin enema my be both diagnostic and therapeutic, so it is the obvious first choice. Afterwards confirm CF disease. Treatment: If unsuccessful, surgery may be needed as well as treating other manifestations of the CF.
A three week old baby has had “trouble feeding” and it is not quite growing well. he now has bilious vomiting and is brought in for evaluation. X-Ray shows a classical “double bubble”, along with normal looking gas pattern in the rest of the bowel. Diagnosis? Management?
What is it? Malrotation.
Treatment: contrast enema to verify the malrotation and emergency surgery.
A 3 week old first-born, full term baby boy began to vomit three days ago. The vomiting is projectile, has no bile in it, follows each feeding and the baby is hungry and eager to eat again after he vomits. He looks somewhat dehydrated and has visible gastric peristaltic waves and a palpable “OLIVE size” mass in the right upper quadrant. Diagnosis? Management?
What is it? Hypertrophic pyloric stenosis.
Management: Check electrolytes: hypokalemic, hypochloremic metabolic alkalosis may have developed. Correct it, rehydrate and do Ramsted Pyloromyotomy.
An 8 week old baby is brought in because of persistent, progressively increasing jaundice. The bilirubin is significantly elevated and about two thirds of it is conjugated, direct bilirubin. Ultrasound rules out extrahepatic masses, serology is negative for hepatitis and sweat test is normal. Diagnosis? Management?
What is it? Biliary atresia.
Management: HIDA scan, percutaneous liver biopsy and exploratory laparotomy.
A two month old baby boy is brought in because of chronic constipation. The kid has abdominal distention, and plain X-Rays show gas in dilated loops of bowel throughout the abdomen. Rectal exam is followed by expulsion of stool and flatus, with remarkable improvement of the distention. Diagnosis? Testing to Confirm?
What is it? Hirschsprungs’s disease. Testing? Barium enema will define the normal-looking aganglionic distal colon and the abnormal-looking thickness biopsy of the rectal mucosa.
A 9 month old, chubby, healthy looking little boy has episodes of colicky abdominal pain that make him double up and squat. The pain lasts for about one minute, and the kid looks perfectly happy and normal until he gets another colick. Physical exam shows a vague mass on the right side of the abdomen, an “empty” right lower quadrant and currant jelly stools. Diagnosis? Management?
What is it? Intussusception.
Management: Barium enema is both diagnostic and therapeutic in most cases. It should be your first choice. Exploratory laparotomy and manual reduction will be needed if not sucessfull.
A one year old baby is referred to the University Hospital for treatment of a subdural hematoma. In the admission examination it is noted that the baby has retinal hemorrhages. OR A three year old girl is brought in for treatment of a fractured humerus. The mother relates that the girl fell from her crib. X-Rays show evidence of other older fractures at various stages of healing in different bones. OR A one year old child is brought in with second degree burns of both buttocks. The stepfather relates that the child fell into a hot tub.
What are these? child abuse. Notify the proper authorities.
A 7 year old boy passes a large bloody bowel movement. Diagnosis? Testing?
What is it? Meckel’s diverticulum. Testing:
Do a radioisotope scan looking for gastric mucosa in the lower abdomen.
A 15 year old girl has a round, 1 cm. cystic mass in the midline of her neck at the level of the hyoid bone. When the mass is palpated at the same time that the tongue is pulled, there seems to be a connection between the two. The mass has been present for at least 10 years, but only recently bothered the patient because it got infected and drained some pus. Diagnosis? Management?
What is it? Thyroglossal duct cyst. Management: Sistrunk operation (removal of the mass and the track to the base of the tongue, along with the medial segment of the hyoid bone).
An 18 year old woman has a 4cm., fluctuant round mass on the side of her neck, just beneath and in front of the sternocleidomastoid. She reports that is has been there at least 10 years, although she thinks that it has become somewhat larger in the last year or two. A CT scan shows the mass to be cystic. Diagnosis? Management?
What is it? A branchial cleft cyst. Management: Elective surgical removal.
A 6 year old child has a mushy, fluid filled mass at the base of the neck, that has been noted for several years. The mass is about 6 cm. in diameter, occupies most of the supraclavicular area and seems by physical exam to go deeper into the neck and chest. Diagnosis? Management?
What is it? Cystic hygroma
Management: Get a CT scan to see how deep this thing goes. They can extend down into the chest and mediastinum. Surgical removal.
A 22 year old lady notices an enlarged lymph node in her neck. The node is in the jugular chain, measures about 1.5 cm, is not tender, and was discovered by the patient yesterday. The rest of the history and physical exam are unremarkable. Diagnosis? Management?
What is it? It’s large lymph node, either inflammatory or neoplastic
Management: before tests, Schedule the patient to be rechecked in three weeks. If the node has gone away by then, it was inflammatory and nothing further is needed. If it’s still there, it could be neoplastic and something needs to be done.
A 22 year old lady seeks help regarding an enlarges lymph node in her neck. The node is in the jugular chain, measures about 2cm, is firm, not tender, and was discovered by the patient six weeks ago. There is a history of low grade fever and night sweats for the past three weeks. Physical examination reveals enlarged lymph nodes in both axillas and in the left groin. Diagnosis? Management?
What is it? Lymphoma most likely. Management: Tissue diagnosis with FNA of the available nodes, but eventual node biopsy will be needed to establish not only the diagnosis but also the type of lymphoma.
A 72 year old man has 4 cm. hard mass in the left supraclavicular area. The mass is movable, non tender and has been present for three months. The patient has had a 20 pound weight loss in the past two months, but is otherwise asymptomatic. Diagnosis? Testing?
What is it? Malignant mets to a supraclavicular node from a primary tumor below the neck. Testing? Look for the obvious primary tumors: lung, stomach, colon, pancreas, kidney. The node itself will eventually be biopsied.
A 69 year old man who smokes and drinks and has rotten teeth has a hard, fixed, 4 cm. mass in his neck. The mass is just medial and in front of the sternomastoid muscle, at the level of the upper notch of the Thyroid cartilage. It has been there for at least six months, and it is growing. Diagnosis? Do not Do What? Management?
What is it? Metastatic squamous cell carcinoma to a jugular chain node, from a primary in the mucosa of the head and neck. Don’t biopsy the node! Management: FNA is OK, but the best answer is to do a triple endoscopy (examination under anesthesia of the mouth, pharynx, larynx, esophagus and tracheobronchial tree).
A 69 year old man who smokes and drinks and has rotten teeth has hoarseness that has persisted for SIX weeks in spite of antibiotic therapy. OR A 69 year old man who smokes and drinks and has rotten teeth has a painless ulcer in the floor of the mouth that has been present for 6 weeks and has not healed. OR A 69 year old man who smokes and drinks and has rotten teeth has unilateral ear ache that has not gone away in 6 weeks. Physical examination shows serious otitis media on that side, but not on the other. Diagnosis? Management?
What is it? squamous cell carcinoma of the mucosa of the head and neck. Management: triple endoscopy to find and biopsy the primary tumor and to look for synchronous second primaries.
A 52 year old man complains of hearing loss. When tested he is found to have unilateral SENSORY hearing loss on one side only. He hoes not engage in any activity (such as sport shooting) that would subject that ear to noise that spares the other side. Diagnosis? Testing?
What is it? acoustic nerve neuroma. How is it diagnosed? MRI looking for the tumor.
A 56 year old man develops slow, progressive paralysis of the facial nerve on one side. It took several weeks for the full blown paralysis to become obvious, and it has been present now for three months. It affects both the forehead as well as the lower face. Diagnosis? Testing?
What is it? Gradual, unilateral nerve paralysis suggests a neoplastic process. Work-up: Gadolinium enhanced MRI.
A 45 year old man presents with a 2 cm. firm mass in front of the left ear, which has been present for four months. The mass is deep to the skin and it is painless. The patient has normal function of the facial nerve. Diagnosis? Testing? Treatment?
What is it? Pleomorphic adenoma (mixed tumor) of the parotid gland. Testing? FNA, recall that parotid masses are never biopsied in the office or under local anesthesia. Treatment: formal superficial parotidectomy via ENT.
A 65 year old man present with a 4 cm. hard mass in front of the left ear, which has been present for six months. The mass is deep to the skin and it is fixed. He has constant pain in the area, and for the past two months has had gradual progression of left facial nerve paralysis. He has rock-hard lymph nodes in the left neck. Diagnosis? Testing?
What is it? Cancer of the parotid gland. Testing? FNA, recall that parotid masses are never biopsied in the office or under local anesthesia. Treatment: formal superficial parotidectomy via ENT.
A two year old boy has unilateral ear ache. OR
A two year old has unilateral foul smelling purulent rhinorrhea. OR A two year old has unilateral wheezing and the lung on that side looks darker on X-Rays (more air) than the other side. Diagnosis? Management?
What is it? foreign body.
Management: Appropriate X-Rays, physical examination or endoscopies and extraction –typically under anesthesia.
A 4 year old child is brought by his mother to the emergency room because “she is sure that he must have swallowed a marble”. The kid was indeed playing with marbles and apparently completely healthy when he was put to bed, but four hours later he had developed inspiratory stridor, a fever of 103 and obvious respiratory distress. The kid is sitting up, leaning forward, drooling at the mouth and looking very sick indeed. Diagnosis? Testing? Management?
What is it? Acute epiglotitis. Testing? diagnosis is confirmed with lateral X-rays of the neck, but be sure experts go with the kid to the X-Ray dept., ready to use bag and mask if needed. Treatment: Then it’s off to the OR for nasotracheal intubation. If bradychardia develops, the kid is in real trouble: atropine will help, but hypoxia is the problem. Along the way, start IV antibiotics for H.Influenzae.
A 45 year old lady with a history of a recent tooth infection shows up with a huge, hot, red, tender, fluctuant mass occupying the left lower side of her face and upper neck, including the underside of the mouth. The mass pushes up the floor of the mouth on that side. She is febrile. Diagnosis? Management?
What is it? Ludwigs’ Angina. (An abscess of the floor of the mouth)
Management: Tracheostomy and incision drainage of the abscess.
A 29 year old lady calls your office at 10 AM with the history that she woke up that morning with one side of her face paralyzed. Diagnosis? Management?
What is it? Bell’s palsy.
Management: start these patients right away on anti-viral medication, often idiopathic and will resolve spontaneously in most cases.
A patient with multiple trauma from a car accident is being attended to in the emergency room. As multiple invasive things are done to him, he repeatedly grimaces with pain. The next day it is noted that he has a facial nerve paralysis on one side. Diagnosis? Treatment?
What is it? – Trauma to the temporal bone can certainly transect the facial nerve, but when that happens the nerve is paralyzed right there and then. Paralysis appearing late is from EDEMA. Treatment: nothing needs to be done.
Your office receives a phone call from Mrs. Rodriguez. You know this middle aged lady very well because you have repeatedly treated her in the past for episodes of sinusitis. In fact, six days ago you started her on decongestants and oral antibiotics for what you diagnoses as frontal and ethmoid sinusitis. Now she tells you over the phone that ever since she woke up this morning, she has been seeing double. Diagnosis? Management?
What is it? Cavernous sinus thrombosis, or orbital cellulitis. Management: Emergency. She needs immediate hospitalization, high dose IV antibiotic treatment and surgical drainage of the paranasal sinuses or the orbit. A CT scan will be needed to guide the surgery.
A 10 year old girl has epistaxis. Her mother says that she picks her nose all the time. Location of bleeding? Management?
What is it? Bleeding from the anterior part of the septum. Management: Phenylephrine spray and local pressure.
An 18 year old boy has epistaxis. The patient denies picking his nose. No source of anterior bleeding can be seen by physical examination. Diagnosis? Management?
What is it? Either septal perforation from cocaine abuse, or posterior juvenile nasopharyngeal angiofibroma.
Management: refer to ENT.
A 72, hypertensive male, on aspirin for arthritis, has a copious nosebleed. His blood pressure is 220/150 when seen in the E.R. He says he began swallowing blood before it began to come out through the front of his nose. Diagnosis? Management?
What is it? epistaxis secondary to hypertension.
Management: Medical Rx. To lower the blood pressure is clearly needed, and may be the option offered in the answers, but getting the ENT people right away should also be part of the equation.
A 57 year old man seeks help for “dizziness”. On further questioning he explains that he gets light headed. OR 57 year old man seeks help for “dizziness”. On further questioning, he explains that the room spins around him. Diagnosis? Management?
What is it? effecting the vestibular apparatus. Management: either symptomatic treatment (meclizine, phenergan, diazepam), or an ENT workup.
During a school physical exam, a 12 year old girl is found to have a heart murmur. She is referred for further evaluation. An alert cardiology fellow recognized that she indeed has a pulmonary flow systolic murmur, but he also notices that she has a FIXED SPLIT second heart sound. A history of frequent colds and upper respiratory infections is elicited. Diagnosis? Testing? Treatment?
What is it? Atrial septal defect. Testing: Echocardiography to establish the diagnosis. Treatment: Surgical closure of the defect.
A three month old boy is hospitalized for ‘failure to thrive”. He has a loud, PANSYSTOLIC heart murmur best heard at the left sternal border. Chest X-Ray shows increased pulmonary vascular markings. Diagnosis? Management:
What is it? Ventricular septal defect. Management: Echocardiography and surgical correction.
A three day old premature baby has trouble feeding and pulmonary congestion. Physical exam shows bounding peripheral pulses and a CONTINUOUS, machinery-like heart murmur. Diagnosis? Management?
What is it? Patent ductus arteriosus. Management: Echocardiography and surgical closure or indomethacin.
A patient known to have a congenital heart defect requires extensive dental work. Prophylaxis For?
antibiotic prophylaxis for subacute bacterial endocarditis.
A 6 year old boy is brought to the U.S. by his new adoptive parents, from an orphanage in Eastern Europe. The kid is small for his age, and has a bluish hue in the lips and tips of his fingers. He has clubbing and spells of cyanosis relieved with SQUATTING. He has a systolic ejection murmur in the left third intercostal space. Chest X-Ray shows a small heart, and diminished pulmonary vascular markings. EKG shows right ventricular hypertrophy. Diagnosis? Management?
What is it? Tetralogy of Fallot. If the kid went home after birth, and later was found to be cyanotic, bet on tetralogy. If he was blue from the moment of birth, bet on transposition.
Management: start with an Echocardiogram as a good diagnostic test with later surgical correction.
A 72 year old man has a history of angina and exertional syncopal episodes. He has a HARSH midsystolic heart murmur best heard at the second intercostal space and along the left sternal border. Diagnosis? Test? Treatment?
What is it? Aortic stenosis.
Test: The diagnostic test is echocardiogram. Treatment: Surgical valvular replacement is indicated if there is a gradient of more than 50 mmHg, or at the first indication of congestive heart failure, angina or syncope.
A 72 year old man has ben known for years to have a wide pulse pressure and a blowing, high-pitched, Diastolic heart murmur best heard at the second intercostal space and along the left lower sternal border with the patient in full expiration. He has had periodic echocardiograms, and in the most recent one there is evidence of beginning left ventricular dilatation. Diagnosis? management?
What is it? Chronic aortic insufficiency. Management: Aortic valve replacement.
A 26 year old drug-addicted man develops congestive heart failure over a short period of a few days. He has a loud, diastolic murmur at the right, second intercostal space. A physical exam done a few weeks ago, when he had attempted to enroll in a detoxification program was completely normal. Diagnosis? Management?
What is it? Acute aortic insufficiency due to endocarditis. Management: Emergency valve replacement, and antibiotics for a long time.
A 35 year old lady has dyspnea on exertion, orthopnea, paroxysmal nocturnal dyspnea, cough and hemoptisis. She has had these progressive symptoms for about 5 years. She looks thin and cachectic, she has atrial fibrillation and a low-pitched, rumbling DIASTOLIC APICAL heart murmur. At age 15 she had rheumatic fever. Diagnosis? Management?
What is it? Mitral stenosis.
Management: Start with echocardiogram. Eventually surgical mitral valve repair.
A 55 year old lady has been known for years to have mitral valve prolapse. She now has developed exertional dyspnea, orthopnea and atrial fibrillation. She has an apical, high pitched, HOLOSYSTOLIC heart murmur that radiates to the axilla and back. diagnosis? Management?
What is it? Mitral regurgitation. Management: Start with the Echocardiogram, eventually surgical repair of the valve (annuloplasty), or possibly valve replacement.
A 55 year old man has progressive, unstable, disabling angina that does not respond to medical management. His father and two older brothers died of heart attacks before the age of 50. The patient stopped smoking 20 years ago, but still has a sedentary life style, is a bit overweight, has type two diabetes mellitus and has high cholesterol. Management?
Management: MI waiting to happen, this man needs a cardiac catheterization to see if he is suitable candidate for coronary revascularization.
A 55 year old man has progressive, unstable, disabling angina that does not respond to medical management. His father and two older brother died of heart attacks before the age of 50. The patient stopped smoking 20 years ago, but still has a sedentary life style, is a bit overweight, has type two diabetes mellitus and has high cholesterol. Cardiac catheterization demonstrates 70% occlusion of three coronary arteries, with good distal vessels. His left ventricular ejection fraction is 65%. Management?
Management: He is lucky. He has good distal vessels (smokers and diabetics often do not) and enough cardiac function left. He clearly needs coronary bypass, and with 3 vessel disease there should be no argument for angioplasty instead of surgery.
On a routine pre-employment physical examination, a chest X-Ray is done on a 45 year old chronic smoker. A “coin lesion” is found in the upper lobe of the right lung. Concern? Next Step?
Concern? Cancer of the lung. Next step: Find an older chest X-Ray to compare, if unchanged check with periodic X-Rays.
A 54 year old man with a 40 pack/year history of smoking gets a chest X-Ray because of persistent cough. A peripheral, 2 cm. “coin lesion” is found in the right lung. A chest X-Ray taken two years ago had been normal. CT scan shows no calcifications in the mass and no enlarged peribronchial or peritracheal lymph nodes. Bronchoscopy and percutaneous needle biopsy have not been able to establish a diagnosis. The man has good pulmonary function and is otherwise in good health. Determining Final Diagnosis? Testing?
Determining diagnosis? With cancer of the lung, there is an interplay of three issues: establishing the diagnosis – which sometimes requires very invasive steps; ascertaining if surgery can be done – i.e.: will the patient still be functional after some lung tissue is removed?; and third, does the surgery have a fair chance to cure him? Here is an example of a man who could stand lung resection (peripheral lesion, good function) and who stands a good change for cure (no node mets). Diagnosis steps should be pushed to the limit. Testing: Start with bronchoscopy and washings, if unrewarding go to percutaneous needle biopsy, and if still unsuccessful go to open biopsy, i.e.: thoracotomy and wedge resection.
A 72 year old chronic smoker with severe COPD is found to have a central, hilar mass on chest X-Ray. Bronchoscopy and biopsy establish a diagnosis of squamous cell carcinoma of the lung. His FEV1 is 100, and a ventilation/perfusion scan shows that 60% of his pulmonary functions comes from the affected lung. Management?
Management: It takes an FEV1 of at least 800 to survive surgery and not be a pulmonary cripple afterwards. If this fellow got a pneumonectomy (which he would need for a central tumor) he would be left with an FEV1 of 440. No way. Don’t do any more tests. He is not a surgical candidate. Pursue chemotherapy and radiation.
A 62 year old chronic smoker has an episode of hemoptysis. Chest X-ray shows a central hilar mass. Bronchoscopy and biopsy establish a diagnosis of squamous cell carcinoma of the lung. His FEV1 is 2200, and a ventilation/perfusion scan shows that 30% of his pulmonary function comes from the affected lung. Management?
Management: This fellow could tolerate a pneumonectomy. Key are that CT scan and mediastinoscopy are in order, to ascertain if surgery has a decent chance to cure him.
A 33 year old lady is undergoing a diagnostic work-up because she appears to have Cushing’s syndrome. Chest X-Ray shows a central, 3 cm. round mass on the right lung. Bronchoscopy and biopsy confirm a diagnosis of small cell carcinoma of the lung. Management?
Management: Radiation and chemotherapy. Small cell lung cancer is not treated with surgery, and thus we have no need to determine FEV1 or nodal status.
A 54 year old right handed laborer notices coldness and tingling in his left hand as well as pain in the forearm when he does strenuous work. What really concerned him, though, is that in the last few episodes he also experienced transitory vertigo, blurred vision and difficulty articulating his speech. Angiogram demonstrates retrograde flow through the vertebral artery. Diagnosis? Management?
What is it? Subclavian steal syndrome. A combination of “claudication of the arm” with posterior brain neurological symptoms Management: already have the angiographic study, so now you are ready for vascular surgery.
A 62 year old man is found on physical exam to have a 6 cm. pulsatile mass deep in the abdomen, between the xiphoid and the umbilicus. Diagnosis? Management?
What is it? Abdominal aortic aneurysm. Management: Needs elective surgical repair.
A 62 year old man has vague, poorly described epigastric and upper back discomfort. He has been found on physical exam to have a 6 cm. pulsatile mass deep in the abdomen, between the xiphoid and the umbilicus. The mass is tender to palpation. Diagnosis? Management?
What is it? Abdominal aortic aneurysm that is beginning to leak.
Management: consultation with the vascular surgeons TODAY.
A 68 year old man is brought to the ER with excruciating back pain that began suddenly 45 minutes ago. He is diaphoretic and has a systolic blood pressure of 90. There is an 8 cm., pulsatile mass palpable deep in his abdomen, between the xiphoid and the umbilicus. Diagnosis? Management?
What is it? Abdominal aortic aneurysm, rupturing right now. What Management? Emergency surgery.
A retired businessman has claudication when walking more than 15 blocks. Management?
Management: Vascular surgery, or angioplasty and stenting are palliative procedures. They do not cure arteriosclerotic occlusive disease. Claudication has an unpredictable course, thus there is no advantage to an “early operation”. This man needs nothing. If he smokes, he should quit.
A 56 year old postman describes severe pain in his right calf when he walks two or three blocks. The pain is relieved by resting 10 or 15 minutes, but recurs if he walks again the same distance. He can not do his job this way, and he does not qualify yet for retirement, so he is most anxious to have this problem resolved. He does not smoke. Management?
Management: Start with Doppler studies to examine extent. Then medical management via lifestyle modification. If for lifestyle necessitiy (i.e. job), If he has significant gradient, arteriogram comes next, followed by bypass surgery or stenting.
A patient consults you because he “can not sleep”. On questioning it turns out that he has pain in the right calf, which keeps him from falling asleep. He relates that the pain goes away if he sits by the side of the bed and dangles the leg. His wife adds that she has watched him do that, and she has noticed that the leg which was very pale when he was lying down becomes deep purple several minutes after he is sitting up. On physical exam the skin of that leg is shiny, there is no hair and there are no palpable peripheral pulses. Diagnosis? Management?
What is it? PVD with Claudication. Management? Definitively he needs the studies to see if vascular surgery could help him.
A 45 year old man shows up in the ER with a pale, cold, pulseless, paresthetic, painful and paralytic lower extremity. The process began suddenly two hours ago. Physical exam shows no pulses anywhere in that lower extremity. Pulse at the wrist is 95 per minute, grossly irregular. Diagnosis? Management?
What is it? - Embolization from in the left atrium.
Management? Emergency surgery with use of Fogarty catheters to retrieve the clot.
A 74 year old man has sudden onset of extremely severe, tearing chest pain that radiates to the back and migrates down shortly after it’s onset. His blood pressure is 220/100, he has unequal pulses in the upper extremities and he has a wide mediastinum on chest X-Ray. Electrocardiogram and cardiac enzymes show that he does not have a myocardial infarction. Diagnosis? Management?
What is it? Dissecting aneurysm of the thoracic aorta. Management: Arteriogram FIRST, but the forces that dissected the vessel plus the force of the dye injection could further shear the aorta, thus study is done with BETA BLOCKERS or IV NITRATES to lower blood pressure. If the aneurysm is in the ascending aorta, emergency surgery will be done. If it is in the descending, ICU intensive therapy for the hypertension will be the preferable option.
A 62 year old right handed man has transient episodes of weakness in the right hand, blurred vision, and difficulty expressing himself. There is not associated headache, the episodes last about 5 or 10 minutes at the most, and they resolve spontaneously. Funduscopic examination reveals highly refractile crystals in the left retinal artery. Diagnosis? Testing? Treatment?
What is it? TIA's around left carotid artery (probably an ulcerated plaque at the left carotid bifurcation).
Testing? The gold standard is still angiogram.
Treatment: Carotid endarterectomy.
A 61 year old man presents with a one year history of episodes of vertigo, diplopia, blurred vision, disarthria and instability of gait. The episodes last several minutes, have no associated headache and leave no neurological sequela. Diagnosis? Management? Treatment?
What is it? Transient ischemic attacks, but now the vertebrals may be involved. Management: choose an arteriogram that examines all the arteries going to the brain: i.e. an aortic arch study. Treatment: Vascular surgery will follow.
A 60 year old diabetic male presents with abrupt onset of right third nerve paralysis and contralateral hemiparesis. There was no associated headache. The patient is alert, but has the neurological deficits mentioned. Diagnosis? Management?
What is it? Neurological catastrophes that begin suddenly and have no associated headache are vascular occlusive, i.e. "a stroke". Management: Vascular surgery in the neck is designed to prevent strokes, not to treat them once they happen. This fellow will get a CT scan to assess the extent of the infarct, and supportive treatment with eventual emphasis on rehabilitation.
A 64 year old black man complains of a very severe headache of sudden onset and then lapses into a coma. Past medical history reveals untreated hypertension and examination reveals a stuporous man with profound weakness in the left extremities. Diagnosis? Management?
What is it? - Neurological catastrophes of sudden onset, with severe headache are vascular hemorrhagic. Management: Again supportive with eventual rehabilitation efforts if he survives. CT scan is the universal first choice to see blood inside the head.
A 39 year old lady presents to the ER with a history of a severe headache of sudden onset that she says is different and worse than any headache she has ever had before. She is given pain medication and sent home. She improves over the next few days, but ten days after the initial visit she again gets a sudden, severe and singular diffuse headache and she returns to the ER. This time she has some NUCHAL RIGIDITY on physical exam. Diagnosis? Testing?
What is it? subarachnoid bleeding from an intracraneal aneurysm.
Testing: start with CT, Angiograms will eventually follow, in preparation for surgery to clip the aneurysm.
A 31 year old nursing student developed persistent headaches that began approximately 4 months ago, have been gradually increasing in intensity and are worse in the MORNINGS. For the past three weeks, she has been having projectile vomiting. Thinking that she may need new glasses, she seeks help from her optometrist, who discovers that she has bilateral papilledema. Diagnosis? Management? Treatment?
What is it? Brain tumor. Neurological processes that develop over a period of a few months and lead to increased intracraneal pressure, spell out tumor. Morning headaches are typical. Management: If given the option, pick MRI as your diagnostic test. If not offered, settle for CT scan. Treatment: Measures to decrease intracraneal pressure while awaiting surgery, include mannitol, hyperventilation, and high dose steroids (decadron).
A 42 year old right handed man has a history of progressive speech difficulties and right hemiparesis for five months. He has had progressively severe headaches for the last two months. At the time of admission he is confused, vomiting, has blurred vision, papilledema and diplopia. Shortly thereafter his blood pressure goes up to 190 over 110, and he develops bradychardia. Diagnosis? Vital Signs Manifestation? Management?
What is it? brain tumor, with localizing signs (left hemisphere, parietal and temporal area). Manifests: the Cushing’s reflux of extremely high intracraneal pressure. Management: emergency to decrease intracraneal pressure while awaiting surgery, include mannitol, hyperventilation, and high dose steroids (decadron).
A 12 year old boy is short for his age, has bitemporal hemianopsia and has a calcified lesion above the sella in X-Rays of the head. Diagnosis? Testing? Treatment?
What is it? Craniopharyngioma.
Testing: MRI. Treatment: proceed with pituitary surgery
A 23 year old nun presents with a history of amenorrhea and galactorrhea of six months duration. She is very concerned that others may think that she is pregnant, and she vehemently denies such a possibility. Diagnosis? Management?
What is it? Prolactinoma.
Management: Every time you suspect a functioning tumor of an endocrine gland, you measure the appropriate hormone. So, here you want a prolactin level. You are also going to do surgery and visualized via prior MRI. If inoperable, Bromocriptine will help.
A 44 year old man is referred for treatment of hypertension. His physical appearance is impressive: he has big, fat, sweaty hands; large jaw and thick lips, large tongue and huge feet. He is also found to have a touch of diabetes. In further questioning he admits to headaches and he produces pictures of himself taken several years ago, where he looks strikingly different. Diagnosis? Management?
What is it? Acromegaly. .
Management: Growth hormone determination, MRI and eventually pituitary surgery.
A 15 year old girl has gained weight and become “ugly”. She shows a picture of herself a year ago, where she was a lovely young lady. Now she has a hairy, red, round face full of pimples; her neck has a posterior hump and her supraclavicular areas are round and convex. She has a fat trunk and thin extremities. She has mild diabetes and hypertension. Diagnosis? Management?
What is it? - Cushing’s. .
Management: AM and PM cortisol levels, dexamethasone suppression test, MRI of the sella and eventually trans-sphenoidal pituitary surgery.
A 27 year old lady presents with a six month history of headaches, visual loss and amenorrhea. The day of admission to the hospital she developed a severe headache, marked deterioration of remaining vision and stupor. Besides the stupor, physical exam is remarkable because her blood pressure is 75/45. Funduscopic examination reveals bilateral pallor of the optic nerves. OR A 55 year old lady is involved in a minor traffic accident where her car was hit sideways by another car that she “did not see” at an intersection. When she is tested further it is recognized that she has bitemporal hemianopsia. Ten years ago she had bilateral adrenalectomies for Cushing’s disease. Diagnosis? Management?
What is it? Nelson’s syndrome. Management: MRI will show the tumor. Trans-nasal, trans-sphenoidal surgery will remove it.
A 42 year old man has been fired from his job because of inappropriate behavior. For the past two months he has gradually developed very severe, “explosive” headaches that are located on the right side, above the eye. Neurologic exam shows optic nerve atrophy on the right, papilledema on the left and anosmia. Diagnosis? Management?
What is it? Brain tumor in the right frontal lobe, this is the Foster-Kennedy syndrome. Management: MRI and neurosurgery.
A 32 year old man complains of progressive, severe generalized headaches, that began three months ago, are worse in the mornings and lately have been accompanied by projectile vomiting. He has lost his upper gaze and he exhibits the physical finding known as “sunset eyes”. Diagnosis? Management?
What is it? Another classic. This tumor is in the pineal gland and if you want the fancy name it is Parinaud’s syndrome.
Management:MRI to start. The neurosurgeons will take care of the rest.
A six year old boy has been stumbling around the house and complaining of severe morning headaches for the past several months. While waiting in the office to be seen, he assumes the knee-chest position as he holds his head. Neurological exam demonstrates truncal ataxia. Diagnosis? Management?
What is it? Tumor of the posterior fosa. Management: MRI, neurosurgery.
A 23 year old man develops severe headache, seizures and projectile vomiting over a period of two weeks. He has low grade fever, and was recently treated for acute otitis media and mastoiditis. Diagnosis? Management?
What is it? Brain abscess. Management: These are seen in CT as well as they would on MRI. Then the abscess has to be resected by the neurosurgeons.
An 18 year old street fighter gets stabbed in the back, just to the right of the midline. He has paralysis and loss of proprioception distal to the injury on the right side, and loss of pain perception distal to the injury on the left side. Diagnosis?
What is it? classical spinal cord hemisection, better known as Brown-Sequard’s.
A patient involved in a car accident sustains a burst fracture of the vertebral bodies. He develops loss of motor function and loss of pain and temperature sensation on both sides distal to the injury, while showing preservation of vibratory sense and position sense. Diagnosis?
What is it? - Anterior cord syndrome.
An elderly man is involved in a rear end automobile collision where he hyperextends his neck. He develops paralysis and burning pain of both upper extremities while maintaining good motor function in his legs. Diagnosis? Management?
What is it? Central cord syndrome. Management: some evidence for high dose corticosteroids.
A 52 year old lady has constant, severe back pain for two weeks. While working on her yard, she suddenly falls and can not get up again. When brought to the hospital she is paralyzed below the waist. Two years ago she had a mastectomy for cancer of the breast. Diagnosis? Management?
What is it? metastatic, extradural tumor from breast leading to a fracture with cord compression or transection. Management: MRI is the best imaging modality for the spinal cord. The neurosurgeons may be able to help if the cord is compressed rather than transected.
A 45 year old male gives a history of aching back pain for several months. He has been told that he had muscle spasms, and was given analgesics and muscle relaxants. He comes in now because of the sudden onset of very severe back pain that came on when he tried to lift a heavy object. The pain is “like an electrical shock that shoots down his leg”, and it prevents him from ambulating. He keeps the affected leg flexed. Straight leg raising test gives excruciating pain. Diagnosis? Management?
What is it? - Lumbar disk herniation, most are L4-L5 or L5-S1. Management: MRI for diagnosis. Bed rest will take care of most of these. Neurosurgical intervention only if there is progressive weakness or sphincteric deficits.
A 79 year old man complains of leg pain brought about by walking and relieved by rest. On further questioning it is ascertained that he has to SIT DOWN OR BEND OVER for the pain to go away. Standing at rest will not do it. Furthermore, he can exercise for long periods of time if he is “hunched over”, such as riding a bike or pushing a shopping cart. He has normal pulses in his legs. Diagnosis? Management?
What is it? Neurogenic claudication. Management: Get MRI. Eventually surgical decompression of this cauda equina.
A 60 year old man complains of extremely severe, sharp, shooting, “like a bolt of lighting”, pain in his face which is brought about by touching a specific area, and which lasts about 60 seconds. His neurological exam is normal, but it is noted that part of his face is unshaven, because he fears to touch that area. diagnosis? Management?
What is it? -Tic doloreaux (trigeminal neuralgia).
Management: Rule out organic lesions with MRI. Treat with ANTICONVULSANTS.
Several months after sustaining a crushing injury of his arm, a patient complains bitterly about constant, burning, agonizing pain that does not respond to the usual analgesic medications. The pain is aggravated by the slightest stimulation of the area. The arm is cold, cyanotic and moist. Diagnosis? management?
What is it? - Causalgia (reflex sympathetic distrophy) Management: A successful sympathetic block is diagnostic, and surgical sympathectomy will be curative.
In the newborn nursery it is noted that a child has uneven gluteal folds. Physical exam of the hips reveals that one of them can be easily dislocated posteriorly with a jerk and a “click”, and returned to normal position with a “snapping”. Diagnosis? Management?
What is it? – Developmental dysplasia of the hip
Management: Abduction splinting. (Don’t order X-Rays in a newborn, because Calcification is still incomplete and you will not see anything).
A 6 year old boy has insidious development of limping with decreased hip motion. He complains occasionally of knee pain on that side. He walks into the office with an antalgic gait. Passive motion of the hip is guarded. Diagnosis? Management?
What is it? Legg-Perthes disease (avascular necrosis of the capital femoral epiphysis). Remember that hip pathology can show up with knee pain. Management: AP and lateral X-Rays for diagnosis. Contain the femoral head within the acetabulum by casting and crutches.
A 13 year old boy complains of pain in the groin ( it could be the knee) and is noted by the family to be limping. He sits in the office with the foot on the affected side rotated towards the other foot. Physical examination is normal for the knee, but shows limited hip motion. As the hip is flexed, the leg goes into external rotation and it can not be rotated internally. Diagnosis? Management?
What is it? slipped capital femoral epiphysis, an orthopedic emergency.
Management: AP and lateral X-Rays for diagnosis. The orthopedic surgeons will pin the femoral head in place.
A little toddler has had the flu for several days, but he was walking around fine until about two days ago. He now absolutely refuses to move one of his legs. He is in pain, holds the leg with the hip flexed, in slight abduction and external rotation and you can not examine that hip he will not let you move it. He has elevated sedimentation rate. Diagnosis? Management?
What is it? orthopedic emergency: septic hip.
Management: Under general anesthesia the hip is aspirated to confirm the diagnosis, and open arthrotomy is done for drainage
A child with a febrile illness but no history of trauma has persistent, severe localized pain in a bone. Diagnosis? management?
What is it? Acute hematogenous osteomyelitis
Management: don’t fall for the X-Ray option. X-Ray will not show anything for two weeks. Do BONE SCAN.
A 12 year old girl is referred by the school nurse because of potential scoliosis. The thoracic spine is curved toward the right, and when the girl bends forward a “hump” is noted over her right thorax. The patient has not yet started to menstruate. Managmement?
Management: Baseline X-Rays are needed to monitor progression. At the onset of menses skeletal maturity is about 80%, so this patient still has a way to go. Bracing may be needed to arrest progression. Pulmonary function could be limited if there is large deformity.
still growing brace unless >40% curvature surgery
A 16 year old boy complains of low grade but constant pain in his distal femur present for several months. He has local tenderness in the area, but is otherwise asymptomatic. X-Rays show a large bone tumor, with “sunburst” pattern and periosteal “onion skinning”. Diagnosis? Management?
What is it? Malignant bone tumor. Either osteogenic sarcoma or Ewing’s sarcoma.
Management: Do not attempt biopsy. Referral is needed, not just to any orthopedic surgeon, but to a specialist on bone tumors.
A 66 year old lady picks up a bag of groceries and her arm snaps broken. Diagnosis? Management?
What is it? A pathologic fracture (i.e: for trivial reasons) means bone tumor, which in the vast majority of cases will be metastatic. Management: Get X-Rays to diagnose this particular broken bone, whole body bone scans to identify other mets, and start looking for the primary. In women, breast. In men, prostate. In heavy smokers, lung.
A 58 year old lady has a soft tissue tumor in her thigh. It has been growing steadily for six months, it is located deep into the thigh, is firm, fixed to surrounding structures and measures about 8 cm. in diameter. Diagnosis of Concern? Testing?
What is it? Soft tissue sarcoma is the concern.
Testing: start with MRI. Leave biopsy and further management to the experts.
A middle aged homeless man is brought to the ER because of very severe pain in his forearm. The history is that he passes out after drinking a bottle of cheap wine and he slept on a park bench for an indeterminate time, probably more than 12 hours. There are no signs of trauma, but the muscles in his forearm are very firm and tender to palpation, and passive motion of his fingers and wrist elicit excruciating pain. Pulses at the wrist are normal. Diagnosis? Management?
What is it? Compartment syndrome. Management; Emergency fasciotomy.
A patient presents to the ER complaining of moderate but persistent pain in his leg under a long leg plaster cast that was applied six hours earlier for an ankle fracture. Diagnosis?
Diagnosis: never give pain medication and do nothing else for pain under a cast. The cast has to come off right away. It may be too tight, it may be compromising blood supply, it may have rubbed off a piece of skin…whatever. Your only acceptable option here is to REMOVE the CAST.
A young man involved in a motorcycle accident has an obvious open fracture of his right thigh. The femur is sticking out through a jagged skin laceration. Management?
Management: open fractures are orthopedic emergencies. This fellow may need to have other problems treated first (abdominal bleeding, intracraneal hematomas, chest tubes, etc), but the open fracture should be in the OR getting cleaned and reduced within six hours of the injury.
A 55 year old lady falls in the shower and hurts her right shoulder. She shows up in the ER with her arm held close to her body, but rotated outwards as if she were going to shake hands. She is in pain and will not move the arm from that position. There is numbness in a small area of her shoulder, over the deltoid muscle. Diagnosis? management?
What is it? Anterior dislocation of the shoulder, with axillary nerve damage.
Management: Get AP and lateral X-Rays for diagnosis. REDUCE
After a grand mal seizure, a 32 year old epileptic notices pain in her right shoulder and she can not move it. She goes to the near-by “Doc in a Box”, where she has X-Rays and is diagnosed as having a sprain and given pain medication. The next day she still has the same pain and inability to move the arm. She comes to the ER with the arm held close to her body, in a “normal” (i.e., not externally rotated, but internally rotated) position. Diagnosis? management?
What is it? Posterior dislocation of the shoulder. Very easy to miss on regular X-Rays. Management: Get X-Rays again but order axillary view or scapular lateral.
A front seat passenger in a car that had a head-on collision relates that he hit the dashboard with his knees, and complains of pain in the right hip. He lies in the stretcher in the ER with the right extremity shortened, adducted, and internally rotated Diagnosis? Management?
What is it? orthopedic emergency: posterior dislocation of the hip. The blood supply of the femoral head is tenous, and delay in reduction could lead to avascular necrosis.
Management: X-Rays and emergency recuction.
A 77 year old man falls in the nursing home and hurts his hip. X-Rays show that he has a displaced femoral neck fracture. Management?
Management: blood supply to the femoral head is compromised in this setting and the patient is better off with a metal prosthesis put in, rather than an attempt at fixing the bone. With intertrochanteric fractures on the other hand, the broken bones can be pinned together and expected to heal.
A football player is hit straight on his right leg and he suffers a posterior dislocation of his knee. Beware what?
Beware: posterior dislocation of the knee can nail the popliteal artery.
Attention to integrity of pulses, arteriogram and prompt reduction are the key issues.
A young recruit complains of localized pain in his tibia after a forced march at boot camp. He is tender to palpation over a very specific point on the bone, but X-Rays are normal Diagnosis?
What is it? Stress fracture.
The lesson here is that stress fractures will not show up radiologically until 2 weeks later. Treat the guy as if he had a fracture (cast) and repeat the X-Ray in 2 weeks.
A man who fell from a second floor window has clinical evidence of fracture of his femur. What image test do you want?
Here are the rules: Always get X-Rays at 90 degrees to each other (for instance, AP and lateral), always include the joints above and below, and if appropriate (this case is) check the other bones that might be in the same line of force (here the lumbar spine).
A healthy 24 year old man steps on a rusty nail at the stables where he works as a horse breeder. Three days later he is brought to the ER moribund, with a swollen, dusky foot, in which one can feel gas crepitation. Diagnosis? Treatment?
What is it? Gas gangrene.
What to do? Tons of IV penicillin and immediate surgical debridement of dead tissue, followed by a trip to the nearest hyperbaric chamber for hyperbaric O2 treatment.
A 55 year old, obese man suddenly develops swelling, redness and exquisite pain at the first metatarsal-phalangeal joint. Diagnosis? Testing? Treatment?
What is it? Gout.
Testing: Diagnosis by serum uric acid determination and identification of uric acid crystals in fluid from the joint. Treatment: colchicine, allopurinol or probenicid.
A 67 year old diabetic has an indolent, unhealing ulcer at the heel of the foot. Diagnosis? Management?
What is it? Ulcer at a pressure point in a diabetic is due to neuropathy, but once it has happened it is unlikely to heal because the microcirculation is poor also. Management: control the diabetes, keep the ulcer clean, keep the leg elevated and be resigned to the thought that you may end up amputating the foot.
A 67 year old smoker with high cholesterol and coronary disease has an indolent, unhealing ulcer at the tip of his toe. The toe is blue, and he has no peripheral pulses in that extremity. Diagnosis? Management?
What is it? - Ischemic ulcers are at the farthest away point from where the blood comes. Management: Doppler studies looking for pressure gradient, arteriogram. Revascularization may be possible, and then the ulcer may heal.
A 44 year old, obese woman has an indolent, unhealing ulcer above her right maleolus. The skin around it is thick and hyperpigmented. She has frequent episodes of cellulitis, and has varicose veins. Diagnosis? Treatment?
What is it? Venous stasis ulcer. Management: Unna boot. Support stocking. Varicose vein surgery.
A 14 year old boy presents in the Emergency Room with very severe pain of sudden onset in his right testicle. There is no fever, pyuria or history of recent mumps. The testis is swollen, exquisitely painful, “high riding”, and with a “horizontal lie”. The cord is not tender. Diagnosis? management?
What is it? Testicular torsion, a urological emergency. Management: Emergency surgery to save the testicle.
A 24 year old man presents in the emergency room with very severe pain of recent onset in his right scrotal contents. There is fever of 103 degree and pyuria. The testis is in the normal position, and it appears to be swollen and exquisitely painful. The cord is also very tender. Diagnosis?
What is it? Acute epididimitis. Management: Epididimitis is not a surgical emergency. All he needs is antibiotic therapy. If unsure if torsion or epididymitis, pick a sonogram to rule out torsion before you choose the non-surgical option.
A 72 year old man is being observed with a ureteral stone that is expected to pass spontaneously. He develops chills, a temperature spike to 104 and flank pain. Diagnosis? management?
What is it? - Obstruction of the urinary tract alone is bad. Infection of the urinary tract alone is bad. But the combination of the two is horrible: a true urological emergency. That’s what this fellow has.
Management: Massive intravenous antibiotic therapy, but the obstruction must also be relieved right now. In a septic patient stone extraction would be hazardous, thus the option in addition to antibiotics would be decompression by ureteral stent or percutaneous nephrostomy.
An adult female relates that five days ago she began to notice frequent, painful urination, with small volumes of cloudy and malodorous urine. For the first three days she had no fever, but for the past two days she has been having chills, high fever, nausea and vomiting. Also in the past two days she has had pain in the right flank. She has had no treatment whatsoever up to this time. Diagnosis? Management?
What is it? Pyelonephritis.
Management: Urinary tract infections should not happen in men or in children, and thus they should trigger a work-up looking for a cause. Women of reproductive age on the other hand, get cystitis all the time and they are treated with appropriate antibiotics without great fuss. However, when they get flank pain and septic signs it’s another story. This lady needs hospitalization, IV antibiotics and at least a Sonogram to make sure that there is no concomitant obstruction.
A 62 year old male presents with chills, fever, dysuria, urinary frequency, diffuse low back pain and an exquisitely tender prostate on rectal exam. Diagnosis? Treatment? What should you not do?
What is it? Acute bacterial prostatitis. Management: The treatment for this man is intuitive: he needs I.V. antibiotics. You should not do any more rectal exams or any vigorous prostatic massage. Doing so could lead to septic shock.
You receive a call from a patient at 3:00 AM. His regular urologist retired five years ago, and he has not sought a replacement. At about 11:00 PM last night, the patient injected himself with papaverine directly into the corpora, as he had been instructed to do for his chronic, organic impotence. He achieved a satisfactory erection and had intercourse, but the erection has not gone away and he still has it at this time. Diagnosis? Management?
What is it? Priapism is another urological emergency. Management: Continued erection beyond four hours begins to damage the corpora. He needs emergency injection of an ALPHA AGONIST (phenylephrine, epinephrine or terbutaline) into the corpora. Once the crisis is over, the patient has to be switched from papaverine to Prostaglandin E1, which in now the agent of choice to achieve erection because it is less likely to produce priapism.
You are called to the nursery to see an otherwise healthy looking newborn boy because he has not urinated in the first 24 hours of life. Physical exam shows a big distended urinary bladder. Diagnosis? Management?
What is it? This represents some kind of obstruction. First look at the meatus, it could be simple meatal stenosis. If it is not, posterior urethral valves is the best bet. Management: Drain the bladder with a catheter (it will pass through the valves). Voiding cystourethrogram for diagnosis, endoscopic fulguration or resection for treatment.
A bunch of newborn boys are lined up in the nursery for you to do circumcisions.
You notice that one of them has the urethral opening in the ventral side of his penis, about mid-way down the shaft. Diagnosis?
What is it? Hypospadias.
DO NOT DO THE circumcision. The foreskin may be needed later for reconstruction when the hypospadias is surgically corrected.
A 7 year old child falls off a jungle gym and has minor abrasions and contusions. When checked by his pediatrician, a urinalysis shows microhematuria. Diagnosis? Management?
What is it? Hematuria from the trivial trauma in kids means congenital anomaly of some sort. Management: start with sonogram. IVP may be needed later.
A 9 year old boy gives a history of three days of burning on urination, with frequency, low abdominal and perineal pain, left flank pain and fever and chills for the past two days. Diagnosis? Management?
What is it? Little boys are not supposed to get UTI's, A congenital anomaly has to be ruled out. Management: treat the infection of course, but also do sonogram right away to begin the work up.
A mother brings her 6-year-old girl to you because “ she has failed miserably to get proper toilet training”. On questioning you find out that the little girl perceives normally the sensation of having to void, voids normally and at appropriate intervals, but also happens to be wet with urine all the time. Diagnosis? Management?
What is it? low implantation of one ureter.
Management: If the vignette did not include physical exam, that would be the next step, which might show the abnormal ureteral opening. Often physical exam does not reveal the anomaly, and imaging studies would be required (start with IVP). Surgical repair will follow.
A 16 year old boy sneaks out with his older brother’s friends, and goes on a beer-drinking binge for the first time in his life. He shortly thereafter develops colicky FLANK pain. Diagnosis? Management?
What is it? Ureteropelvic junction obstruction.
Management: Start with ultrasound (sonogram). Repair will follow.
A 62 year old male known to have normal renal function reports an episode of gross, painless hematuria. Further questioning determines that the patient has total hematuria rather than initial or terminal hematuria. Diagnosis? Management?
What is it? - The blood is coming anywhere from the kidneys to the bladder, rather than the prostate or the urethra. Either infection or tumor can produce hematuria. In older patients without signs of infection cancer is the main concern, and it could be either renal cell carcinoma or transitional cell cancer of the bladder.
Management: “gold standard-first study” is IVP (Intravenous pyelogram). It’s main contraindication is poor renal function. If normal, the next step is cystoscopy.
A 70 year old man is referred for evaluation because of a triad hematuria, flank pain and a flank mass. He also has hypercalcemia, erythrocytosis and elevated liver enzymes. Diagnosis? management?
What is it? renal cell carcinoma Management: IVP first and CT scan next would be the standard sequence.
A 61 year old man presents with a history of hematuria. Intravenous pyelogram shows a renal mass, and sonogram shows it to be solid rather than cystic. CT scan shows a heterogenic, solid tumor. Diagnosis?
What is it? renal cell carcinoma.
A 55 year old, chronic smoker, reports three instances in the past two weeks when he has had painless, gross, total hematuria. In the past two months he has been treated twice for irritative voiding symptoms, but has not been febrile and urinary cultures have been negative. Diagnosis? Management?
What is it? Most likely bladder cancer. Management: With this very complete presentation some urologist would go for the cystoscopy first, but the standard sequence of IVP first and cystoscopy next is the only correct answer for an exam.
A 59 year old black man has a rock-hard, discrete, 1.5 cm. nodule felt in his prostate during a routine physical examination. Diagnosis? Management?
What is it? Cancer of the prostate. Management: Trans-rectal needle biopsy. Eventually surgical resection after the extent of the disease has been established.
A 62 year old gentleman had a radical prostatectomy for cancer of the prostate three years ago. He now presents with widespread bony pain. Bone scans show metastasis throughout the entire skeleton, including several that are very large and very impressive. Management?
What to do? dramatic palliation cn be obtained with orchiectomy, although it will not be long-lasting (one or two years only). An expensive alternative is luteinizing hormone-releasing hormone agonists, and another option is antiandrogens (flutamide).
An 82 year old gentleman who has congestive heart failure and COPD is told by his primary care physician that his level of prostatic specific antigen (PSA) is abnormally high. The gentleman has seen ads in the paper for sonographic examinations of the prostate, and he has one done. The examination reveals a prostatic nodule, which on trans-rectal biopsy is proven to be carcinoma of the prostate. The man is completely asymptomatic as far as this cancer is concerned. He has not evidence of metastasis either. Diagnosis?
What is it? After a certain age, most men get prostatic cancer…but die of something else. As a rule, asymptomatic prostatic cancer is not treated after age 75.
A 25 year old man presents with a painless, hard testicular mass. Diagnosis? Managment?
What is it? - Testicular cancer. Management: This will sound horrible, but here is a disease where we shoot to kill first…and ask questions later. The diagnosis is made by performing a radical orchiectomy by the inguinal route. That irreversible, drastic step is justified because testicular tumors are almost never benign. Beware of the option to do a trans-scrotal biopsy: that is a definitive no-no. further treatment will include lymph node dissection in some cases ( and platinum-based chemotherapy. Serum markers are useful for follow up: alfa-fetoprotein and beta-HCG, and they have to be drawn before the orchiectomy (but they do not determine the need for the diagnostic orchiectomy, that still needs to be done).
A 25 year old man is found on a pre-employment chest X-Ray to have what appears to be a pulmonary metastasis from an unknown primary tumor. Subsequent physical examination discloses a hard testicular mass, and the patient indicates that for the past six months he has been losing weight for no obvious reason. Diagnosis? Treatment?
What is it? testicular cancer Treatment: responds to chemotherapy, that treatment is undertaken regardless of the extent of the disease when first diagnosed.
A 60 year old man shows up in the ER because he has not been able to void for the past 12 hours. He wants to, but can not. On physical exam his bladder is palpable half way between the pubis and the umbilicus, and he has a big, boggy prostate gland without nodules. He gives a history that for several years now, he has been getting up four or five times a night to urinate. Because of a cold, two days ago he began taking anthihistaminies, using “nasal drops”, and drinking plenty of fluids. Diagnosis? management?
What is it? Acute urinary retention, with underlying benign prostatic hypertrophy.
Management: Indwelling bladder catheter, to be left in for at least 3 days. Long term therapy includes many options, best are probably long-term alpha-blockers for symptomatic relief, or some form of prostatic resection.
On the second post-operative day after surgery for repair of bilateral inguinal hernias, the patient reports that he “can not hold his urine”. Further questioning reveals that every few minutes he urinates a few cc’s of urine. On physical examination there is a large palpable mass arising from the pelvis and reaching almost to the umbilicus. Diagnosis? management?
What is it? Acute urinary retention with overflow incontinence. Management: Indwelling bladder catheter.
A 42 year old lady consults you for urinary incontinence. She is the mother of five children and ever since the birth of the last one, seven years ago, she leaks a small amount of urine whenever she sneezes, laughs, gets out of a chair or lifts any heavy objects. She relates that she can hold her urine all through the night without any leaking whatsoever. Diagnosis? Management?
What is it? Stress incontinence. Management: Surgical repair of the pelvic floor.
A 72 year old man who in previous years has passed a total of three urinary stones is now again having symptoms of ureteral colic. He has relatively mild pain that began six hours ago, and does not have much in the way of nausea and vomiting. X-Rays show a 3mm. Ureteral stone just proximal to the ureterovesical junction. management?
Management: Urologists have an array of options to treat stones, including laser beams, shock waves, ultrasonic probes, baskets for extraction…etc, but there is still a role for watching and waiting. This man is a good example: small stone, almost at the bladder. Give him time, medication for pain, and plenty of fluids, and he will probably pass it.
A 54 year old lady has a severe ureteral colic. IVP shows a 7mm. Ureteral stone at the ureteropelvic junction. Management?
Management: whereas a 3mm. Stone has a 70% chance of passing, a 7mm. Stone only has a 5% probability of doing so. This one will have to be smashed and retrieved. Best option among answers offered would be shock-wave-lithrotripsy).
A 33 year old man has urgency, frequency, and burning pain with urination. The urine is cloudy and malodorous. He has mild fever. On physical exam the prostate is not warm, boggy or tender. Diagnosis? Treatment?
Diagnosis? This infection needs to be treated, so ask for urinary cultures and start antibiotics…but also start a urological work-up. Do not start with cystoscopy (do not instrument an infected bladder, you could trigger septic shock). Start with either IVP or sonogram
A 72 year old man consults you with a history for that for the past several days he has noticed that bubbles of air come out along with the urine when he urinates. He also gives symptoms suggestive of mild cystitis. Diagnosis? management?
What is it? Pneumaturia due to a fistula between the bowel and the bladder.
Management: Intuitively you would think that either cystoscopy or sigmoidoscopy would verify the diagnosis, but real life does not work that way: they seldom show anything. Contrast studies (cystogram or barium enema) are also typically unrewarding. The test to get is CT scan. Because ruling out cancer of the sigmoid is important, the sigmoidoscopic exam would be done at some point, but not as the first test. Eventually surgery will be needed.
A 32 year old man has sudden onset of impotence. One month ago he was unexpectedly unable to perform with his wife after an evening of heavy eating and heavier drinking. Ever since then he has not been able to achieve an erection when attempting to have intercourse with his wife, but he still gets nocturnal erections and can masturbate normally. Diagnosis? management?
What is it? psychogenic impotence; young man, sudden onset, partner-specific. Organic impotence is typically older, of gradual onset and universal.
Management: Curable with psychotherapy if promptly done. (It will become irreversible after two years).
A 44 year old man has unequivocal signs of multiple liver metastasis, but no primary tumor has been identified by multiple diagnostic studies of the abdomen and chest. The only abnormality in the physical exam is a glass eye, and history of enucleation for a unknown tumor.What was the primary?
Primary? malignant melanoma
A one year old child is suspected of having strabismus. You verify that indeed the corneal reflection from a bright light in your examining room comes from different places from each of his eyes. Concern? Treatment?
Concern: permanent cortical blindness during development; Treatment: surgical correction ASAP
The Causes of Cyanosis in Children.
Cyanotic kids could have any of the 5 conditions that begin with the latter “T”: Tetralogy (delayed) or Transposition (immediate) of the great vessels, which are common; or Truncus arteriosus, Total anomalous pulmonary venous connection or Tricuspid atresia, which are rare.
What is Nelson's Syndrome?
Years ago, before imaging studies could identify pituitary microadenomas, patients with Cushing’s were treated with bilateral adrenalectomy instead of pituitary surgery. In some of those patients the pituitary microadenoma kept on growing and eventually gave pressure symptoms, i.e. bitemporal heminopsia. That is Nelson’s syndrome.